Übungen zur Vorlesung „Mathematische Grundlagen“

Werbung
Übungen zur Vorlesung „Mathematische
Grundlagen“
3. Auflage 2014
2
Inhaltsverzeichnis
Inhaltsverzeichnis
2
A Übungsaufgaben
A.1 Übungen Logik . . . . . . . . . . . . .
A.2 Übungen Grundlagen . . . . . . . . . .
A.3 Übungen Kombinatorik . . . . . . . . .
A.4 Übungen Wahrscheinlichkeitsrechnung
A.5 Übungen Vektoralgebra . . . . . . . . .
A.6 Übungen Matrizenalgebra . . . . . . .
A.7 Übungen Determinanten . . . . . . . .
3
.
.
.
.
.
.
.
.
.
.
.
.
.
.
.
.
.
.
.
.
.
.
.
.
.
.
.
.
.
.
.
.
.
.
.
.
.
.
.
.
.
.
.
.
.
.
.
.
.
.
.
.
.
.
.
.
.
.
.
.
.
.
.
.
.
.
.
.
.
.
.
.
.
.
.
.
.
.
.
.
.
.
.
.
.
.
.
.
.
.
.
.
.
.
.
.
.
.
.
.
.
.
.
.
.
5
5
32
61
71
76
78
81
4
INHALTSVERZEICHNIS
Anhang A
Übungsaufgaben
Ziel der Übungen ist, den Stoff aus der Vorlesung zu vertiefen. Durch diese Vertiefung soll ein besseres Verständnis für den Stoff selbst gegeben werden. Die nachfolgenden Aufgaben sind alle Prüfungsfragen aus ehemaligen Klausuren. Zum
Lernen sind jedoch andere Aufgaben wichtig, die Sie sich selbst stellen sollten,
um den Stoff zu erlernen. Ziel von Prüfungsaufgaben ist, erlerntes Wissen oder
Kompetenzen abzufragen. Ziel anderer Aufgaben kann die Ergründung fachlicher
Themen sein.
A.1
Übungen Logik
Aufgabe A.1 (Boole’sche Algebra)
1. Begriffsklärung (? Punkte)
Zeigen Sie: In einer Boole’schen Algebra (B; ∗; +; K) kann kein Element
komplementär zu sich selbst sein.
Also zeigen Sie: ¬(∃b ∈ B : b−1 = b)
2. Begriffsklärung (? Punkte)
Zeigen Sie: Eine Boole’sche Algebra (B; ∗; +; K) mit fünf Elementen, d.h.
|B| = 5, existiert nicht. Verwenden Sie bei der Argumentation die Ergebnisse aus der vorangegangenen Aufgabe.
3. Analyse (? Punkte)
Für welche Belegungen (x1 , x2 , x3 , x4 ) ∈ B = {0, 1} nimmt der Term
−1
T (x1 , x2 , x3 , x4 ) = (x1 + x2 ) ∗ (x−1
2 + x3 ) ∗ (x1 + x4 )
den Wert 1 an ?
4. Modellierung (? Punkte)
Ein Term T (x1 , x2 , x3 , x4 ) nehme den Wert 0 genau dann an, wenn wenigstens drei Variablen mit 1 belegt sind. Stellen Sie die Wahrheitstafel (16
5
6
ANHANG A. ÜBUNGSAUFGABEN
Zeilen!) auf und geben Sie dann T in der kanonischen konjunktiven Normalform an. Warum eignet sich diese Normalform in diesem Fall besser als
die kanonische disjunktive Normalform ?
Eine kanonische konjunktive Normalform ist eine konjunktive Normalform,
in der in jedem Konjunktionsglied alle verwendeten Variablen (oder deren
Inversen) vorkommen (hier also x1 , x2 , x3 und x4 oder deren Inversen).
In der Boole’schen Algebra wird dies auch als Maxterm (Volldisjunktion)
bezeichnet.
5. Modellierung (? Punkte)
Die dreistellige Schwellenfunktion s2 : {0, 1}3 → {0, 1} mit Schwelle 2 ist
definiert durch:
1 falls x0 + x1 + x2 ≥ 2
s2 (x0 , x1 , x2 ) :=
0 falls x0 + x1 + x2 < 2
Geben Sie eine Boole’sche Formel in disjunktiver Normalform an, die s2
darstellt.
6. Modellierung (? Punkte)
Der Leistungsnachweis für ein Lehrfach gelte als erbracht, wenn die Bedingungen des 1. oder 2. Falles erfüllt sind:
1. Fall Klausurarbeit bestanden (x1 = 1) und 80% der abgegebenen Übungsaufgaben richtig gelöst (x2 = 1) und wenigstens eine der beiden folgenden Bedingungen erfüllt ist:
• 2/3 aller Übungsaufgaben abgegeben (x3 = 1)
• mündliche Prüfung bestanden (x4 = 1)
2. Fall Klausurarbeit nicht bestanden (x1 = 0) und 80% der abgegebenen
Übungsaufgaben richtig gelöst (x2 = 1) und 2/3 aller Übungsaufgaben
abgegeben (x3 = 1) und die mündliche Prüfung bestanden (x4 = 1)
Führen Sie x1 , x2 , x3 , x4 als Schaltvariable ein und stellen Sie für die Schaltfunktion (x1 , x2 , x3 , x4 ) 7→ T die vollständige Wertetafel auf, wobei T = 1
genau dann wenn der Leistungsnachweis erbracht ist. Wie lautet der Term
T (x1 , x2 , x3 , x4 ) in einer minimalen Form, d.h. welche Normalform ist hier
minimal bzgl. ihrer Anzahl an Termen ?
7. Äquivalenz (? Punkte)
In einer Boole’schen Algebra sind die Aussagen a ∧ b = a, a ∨ b = b,
a−1 ∨ b = 1 und a ∧ b−1 = 0 äquivalent in folgender Weise: aus jeder
Gleichung lassen sich die drei anderen herleiten. Beweisen Sie, dass diese
Aussagen äquivalent sind!
Tipp: Nehmen Sie zunächst die Gültigkeit der ersten Gleichung an und
beweisen Sie die zweite Gleichung etc.
A.1. ÜBUNGEN LOGIK
7
8. Normalformen (? Punkte)
Gegeben sei nachfolgende Wahrheitstafel. Geben Sie die disjunktive und konjunktive kanonische Normalform für T (x1 , x2 , x3 ) an.
x1
0
0
0
0
1
1
1
1
x2
0
0
1
1
0
0
1
1
x3
0
1
0
1
0
1
0
1
T
0
1
1
0
1
0
0
1
9. DNF (? Punkte)
Gegeben sei der boole’sche Term q zur Boole’schen Algebra ({0, 1}, ∗, +, −1 }.
q ≡ (x1 ∗ (x2 + x3 )−1 ) + (((x1 ∗ x2 ) + x−1
3 ) ∗ x1 )
(a) Erstellen Sie eine DNF von q mittels Wahrheitstafel.
(b) Erstellen Sie eine DNF von q mittels detaillierter Transformation unter Angabe der verwendeten Gesetze (fundamentale Äquivalenzen wie
in Aussagenlogik).
(c) Zeigen Sie die semantische Äquivalenz beider Formeln (aus den Teilaufgaben a) und b)) durch Transformation.
Tipp: versuchen Sie dazu, die gefundene DNF jeweils zu minimieren.
10. Normalformen (? Punkte)
Welche der folgenden vier Boole’schen Ausdrücke sind semantisch äquivalent ? Versuchen Sie dazu alle Terme zunächst auf eine der Normalformen
(KNF oder DNF) zu bringen. Bei der Transformation verweisen Sie unbedingt auf alle verwendeten Regeln aus dem Tipp, etwa ((c−1 )−1 ) + a ≡(e).(g)
a + c. Beachten Sie: da im Allgemeinen B 6= {0, 1} können Sie hier nicht
mit einer Wahrheitstafel arbeiten!
(a) ((a + b−1 )−1 + (a−1 ∗ b)) + c−1
(b) ((a ∗ c) + ((b + 0)−1 ∗ c))−1
(c) ((a ∗ c)−1 ∗ (a−1 + c−1 )) ∗ (b + c−1 + b)
(d) (a−1 + (b ∗ c))−1 + (c ∗ (b + c))
Tipp: Die fundamentalen Äquivalenzen der Boole’schen Algebra sind im
Folgenden aufgeführt. Die Grundmenge wird mit B notiert, die EIN S ∈ B
mit 1 und die N U LL ∈ B mit 0.
8
ANHANG A. ÜBUNGSAUFGABEN
(a) Absorption: ∀a, b ∈ B : a + (a ∗ b) ≡ a und a ∗ (a + b) ≡ a
(b) Assoziativität: ∀a, b, c ∈ B : a + (b + c) ≡ (a + b) + c und
a ∗ (b ∗ c) ≡ (a ∗ b) ∗ c
(c) DeMorgan: ∀a, b ∈ B : (a + b)−1 ≡ a−1 ∗ b−1 und (a ∗ b)−1 ≡ a−1 + b−1 .
(d) Distributivität: ∀a, b, c ∈ B : a+(b∗c) ≡ (a+b)∗(a+c) und a∗(b+c) ≡
(a ∗ b) + (a ∗ c)
(e) Doppeltes Komplement: ∀a ∈ B : (a−1 )−1 ≡ a
(f ) Idempotenz: ∀a ∈ B : a + a ≡ a ≡ a ∗ a
(g) Kommutativität: ∀a, b ∈ B : a + b ≡ b + a und a ∗ b ≡ b ∗ a
(h) Komplement: ∀a ∈ B : a + a−1 ≡ 1 und a ∗ a−1 ≡ 0
(i) Neutrale Elemente: ∀a ∈ B : a + 0 ≡ a ≡ a ∗ 1 und a ∗ 0 ≡ 0 sowie
a+1≡1
Aufgabe A.2 (Aussagenlogik)
1. Begriffsklärung (? Punkte)
Erklären Sie die drei wesentlichen Bestandteile eines logischen Systems:
Syntax, Semantik und Kalkül. Erklären Sie insbesondere bei der Semantik,
was unter einem Modell verstanden wird.
2. Aussagen (? Punkte)
Tatsachen sind beweisbar und damit Aussagen (im logischen Sinne), Meinungen muss man ausdiskutieren und sind daher keine Aussagen (im logischen Sinne). Welche Sätze sind hier Tatsachen bzw. Meinungen? (Begründung unbedingt angeben!)
(a) „Geld macht nicht glücklich, aber es beruhigt.“
(b) „Manche behaupten, der Mensch stamme von Adam und Eva ab.“
(c) „Rauchen gefährdet die Gesundheit.“
(d) „Politik ist ein schmutziges Geschäft.“
(e) „Es gibt Menschen, die übersinnliche Kräfte haben“.
(f ) „Manche Zeitungsmeldungen sind falsch.“
3. Aussagen (? Punkte)
Seien i, j ∈ Z ganze Zahlen. Bestimmen Sie, ob die folgenden Formeln Tautologien, kontigente Formeln oder Widersprüche sind! Bilden Sie außerdem
für jede Formel das Gegenteil (im Sinne der logische Negation)!
(a) ∀i ≤ 0 : i < j.
(b) ∀i ≤ 0 : ∃j < 0 : i < j.
(c) ∀i < 0 : ∃j < 0 : i ≤ j.
A.1. ÜBUNGEN LOGIK
9
(d) ∃j < 0 : ∀i < 0 : i ≤ j.
(e) i2 ≤ 0.
(f ) i2 ≤ i2 − 1.
(g) (i + j)2 = i2 + 2 ∗ i ∗ j + j 2 .
4. Aussagen (? Punkte)
Modellieren Sie die folgenden Sätze möglichst adäquat als aussagenlogische
Formeln. In schwierigen Fällen begründen Sie bitte kurz die Wahl der Junktoren.
(a) Die Situation ist hart, aber nicht hoffnungslos.
(b) Genau dann, wenn sich weder eine Diskette noch eine CD im Laufwerk
befindet, kann der Computer von der Festplatte booten.
(c) An Weihnachten schneit es, es sei denn, man ist in Bielefeld oder in
der Karibik.
(d) Wenn die Bayern alle Spiele gewinnen, dann werden sie von allen
gehasst, aber wenn sie nicht alle Spiele gewinnen, werden sie von niemandem geliebt.
Prüfen Sie folgendes mittels Wahrheitstafel nach: Aus der Modellierung des
letzten Satzes ((d)) folgt die Aussage „Wenn die Bayern von jemandem geliebt werden, dann werden sie von allen gehasst.“. Diskutieren Sie das Ergebnis.
5. Aussagen (? Punkte)
Welche der folgenden Aussagen sind wahr bzw. falsch ? Begründen Sie bitte
Ihre Antworten! Gehen Sie davon aus, das Lügner hier immer lügen und
nie die Wahrheit sagen.
(a) Eine Klauselmenge M ist erfüllbar gdw. jede endliche Teilmenge von
M erfüllbar ist.
(b) Wenn 2 + 2 = 5 gilt, dann schreiben wir das Jahr 1984.
(c) Das Bibelzitat1 „Ein Kreter sagt: Alle Kreter lügen.“ ist in sich widersprüchlich.
(d) Dieser Satz ist falsch.
1
Brief des Paulus an Titus 1,12
10
ANHANG A. ÜBUNGSAUFGABEN
6. Tautologie (? Punkte)
Welche der folgenden aussagenlogischen Ausdrücke sind Tautologien ? Verwenden Sie Wahrheitstafeln zum Beweis. (x ↔ y ist dabei (x∧y)∨(¬x∧¬y)
und x → y ist ¬x ∨ y)
(a) (x → y) ↔ [(x ∧ ¬y) → y]
(b) (x → y) → [(z ∨ x) → (z ∨ y)]
(c) [(x → y) → (y → z)] → (x → z)
7. Tautologie (? Punkte)
Seien F, G, H aussagenlogische Formeln. Zeigen Sie mit Hilfe der fundamentalen Äquivalenzen:
(a) Ist F → H eine Tautologie, so ist F äquivalent zu F ∧ H.
(b) ((F ∨ H) ∧ (G ∨ ¬H)) → (F ∨ G)
8. Äquivalenz (? Punkte)
Gegeben seinen die folgenden beiden aussagenlogischen Formeln:
• F := (a ∧ b) ∨ (a ∧ c) ∨ (b ∧ c)
• G := (a ∧ b) ∨ (c ∧ ¬(a ↔ b))
Zeigen Sie, dass F und G semantisch äquivalent sind, indem Sie
(a) für beide Formeln eine Wahrheitstafel aufstellen.
(b) G durch die Anwendung der aussagenlogischen fundamentalen Äquivalenzregeln in F überführen.
9. Wahrheitstafel (? Punkte)
Gegeben seinen die folgenden beiden aussagenlogischen Formeln:
• F := (¬(a → b) ∧ c) ↔ c
• G := ¬(c ∧ (a → (b ∨ ¬c)))
(a) Erstellen Sie eine Wahrheitstafel der Formeln F und G ohne vorher
diese zu transformieren!
(b) Sind die Formeln erfüllbar, kontingent und/oder falsifizierbar ?
(c) Sind die Formeln semantisch äquivalent ?
(d) Erstellen Sie für F oder G eine kanonische DNF oder KNF.
10. Begiffe (? Punkte)
Welche der folgenden Aussagen sind wahr ? (Begründung nicht vergessen!)
A.1. ÜBUNGEN LOGIK
11
(a) A und B seien erfüllbar. Dann ist
A ∧ B erfüllbar
A ∨ B falsifizierbar
A → B kontingent
¬A falsifizierbar
(b) A sei erfüllbar und B nicht erfüllbar. Dann ist
A ∧ B ein Widerspruch
A → B ein Widerspruch
A ∨ B erfüllbar
B → A eine Tautologie
(c) A sei keine Tautologie und B erfüllbar. Dann ist
A ∧ B kontingent
¬A ∧ (A ∨ B) falsifizierbar
(¬A) → B erfüllbar
¬A ein Widerspruch
(d) B ist Tautologie, wenn A und A → B Tautologien sind.
11. Wahrheitstafel ( Punkte)
Gegeben sei die aussagenlogische Formel F
F ≡ (¬A ∨ ¬(¬B ∨ ¬A)) → (C ∨ (¬B ↔ A))
(a) Erstellen Sie eine Wahrheitstafel der Formel F ohne vorher diese zu
transformieren!
(b) Ist die Formel erfüllbar und/oder kontingent und/oder falsifizierbar ?
(c) Erstellen Sie eine kanonische DNF oder KNF, je nachdem, welche
kürzer ist.
12. Wahrheitstafel (? Punkte)
Gegeben sei folgende Formel:
H = (A ∧ ¬B ∧ C) ∨ (A ∧ B ∧ ¬C) ∨ (A ∧ ¬B ∧ ¬C)
(a) Bestimmen Sie die Wahrheitstafel der Formel H.
(b) Beschreiben Sie drei unterschiedliche Möglichkeiten, eine semantisch
äquivalente Formel in KNF für die Formel H zu erstellen. Wählen Sie
dann eine dieser Möglichkeiten, um eine semantisch äquivalente Formel in KNF zu erstellen, die aus maximal zwei Klauseln besteht. Weisen Sie die Korrektheit dieser Äquivalenz nach. Begründen Sie kurz,
warum Sie sich für die gewählte Möglichkeit zur Erstellung der KNF
entschieden haben und warum Sie die anderen nicht genommen haben.
(c) Gegeben sei folgende Definition:
12
ANHANG A. ÜBUNGSAUFGABEN
Definition A.1 Sei F eine aussagenlogische Formel. Ein Modell A
heisst kleinstes Modell von F , wenn für alle Modelle B von F und alle
aussagenlogische Variablen a in F gilt: wenn A(a) = 1, dann ist auch
B(a) = 1.
Bestimmen Sie das/die kleinsten Modelle der Wahrheitstafel in Aufgabenteil (a)
13. Äquivalenz (? Punkte)
Es seien A und B mathematische Aussagen. Zeigen Sie durch geeignete
äquivalente Umformungen:
((A ∨ B) → A) ↔ (B → (A ∧ B))
14. Äquivalenz (? Punkte)
Es seien A, B, C und D mathematische Aussagen. Zeigen Sie durch geeignete äquivalente Umformungen (also ohne die Verwendung einer Wahrheitstafel) mit Hilfe des DeMorganschen Gesetzes, der Assoziativität, der
Distributivität und der Kommutativität schrittweise unter Angabe der verwendeten Gesetze, dass folgende beide Aussagen äquivalent sind.
(¬(D → ((A ∧ ¬B) ∧ C)))
((¬A ∧ D) ∨ ((B ∧ D) ∨ (¬C ∧ D)))
Beachten Sie: Das doppelte Komplement kann direkt weggelassen werden.
Die Klammern sind jedoch nur mittels Assoziativität verschiebbar, jedoch
nicht wegzulassen!
15. NAND (? Punkte)
Wir bezeichnen im Folgenden den logischen Junktor nand mittels dem Symbol |.
(a) Untersuchen Sie, ob der |-Junktor assoziativ ist.
(b) Bilden Sie die zur Formel (x1 |x2 )|x3 äquivalente kanonische DNF. Finden Sie mittels Transformation dieser kanonischen DNF eine vereinfachte DNF, die maximal zwei Konjunktionsglieder hat.
16. Semantisch äquivalent (? Punkte)
Seien F, G aussagenlogische Formeln. Zeigen Sie mit Hilfe der fundamentalen Äquivalenzen durch geeignete Transformationen die semantische Äquivalenz (≡) der nachfolgenden Formeln. Verwenden Sie dazu folgende Definitionen: A ↔ B ≡ (A → B) ∧ (B → A) sowie A → B ≡ ¬A ∨ B.
(a) F ↔ ¬G ≡ ¬F ↔ G
(b) ¬F ↔ G ≡ (F ∨ G) ∧ ¬ (F ∧ G)
(c) (F ∨ G) ∧ ¬ (F ∧ G) ≡ (F → ¬G) ∧ (¬F → G)
A.1. ÜBUNGEN LOGIK
13
(d) (F → ¬G) ∧ (¬F → G) ≡ ¬ (F ↔ G)
17. Verschlüsselung (? Punkte)
Im ASCII-Code heißt
GEHEIM 1000111.1000101.1001000.1000101.1001001.1001101 und
SECRETE 1010011.1000101.1000011.1010010.1000101.1010100.1000101.
J.R. Hacker kennt die xor-Funktion und möchte seinem Freund die Nachricht GEHEIM verschlüsselt im Internet übermitteln, damit kein anderer sie
lesen kann. Als Schlüssel verwendet er die ersten sechs Buchstaben des Wortes SECRETE, also SECRET. Diesen Schlüssel hat er seinem Freund bei einem
Treffen schon mal persönlich übergeben.
(a) Stellen Sie die Wahrheitstafel für die Funktion xor auf.
(b) Verschlüsseln Sie nun die Nachricht GEHEIM = x mit dem Schlüssel SECRET = y, indem Sie die einzelnen binären Werte xi , yi ; i ∈
{1, . . . , 42} durch die xor-Funktion verknüpfen, also zi = xi xor yi .
Das Ergebnis ist die binäre Folge z.
(c) Welchen Zeichen in der ASCII-Tabelle entspricht dieser Ausdruck ?
ASCII-Tabellen finden Sie z.B. in den meisten Büchern über die Programmiersprache C.
(d) Entschlüsseln Sie nun die verschlüsselte Nachricht z mit dem Schlüssel
SECRET. Was kommt heraus ?
18. Beweise (? Punkte)
(a) Zeigen Sie mit Hilfe einer Wahrheitstafel, dass das Distributivgesetz
A ∨ (B ∧ C) ≡ (A ∨ B) ∧ (A ∨ C) gilt.
(b) Die Formeln beim Distributivgesetz sind in DNF (linke Seite) und
KNF (rechte Seite) gehalten. Bei einer kanonischen DNF bzw. kanonischen KNF sind bei der jeweiligen konjunktiven bzw. disjunktiven Klausel alle aussagenlogischen Variablen vorhanden. Geben Sie zu
dem Distributivgesetz entweder die kanonische DNF oder die kanonische KNF an, je nachdem, welche am kürzesten ist (am wenigsten
Klauseln beinhaltet).
(c) Zeigen Sie durch geeignete äquivalente Umformungen, dass die Aussage ((A → B) ∧ ((¬A) → B)) → B unabhängig von den Wahrheitswerten von A und B stets wahr ist.
14
ANHANG A. ÜBUNGSAUFGABEN
19. Interpretation (? Punkte)
Gegeben sei die aussagenlogische Formel F = ¬((A ∧ B) ∨ C) und die Belegungen A(A) = 1; A(B) = 1; und A(C) = 0;. Bestimmen Sie A(F ) durch
Anwendung der Definition der Semantik der Aussagenlogik (Belegungsfunktion A).
20. Modellierung (? Punkte)
Es sei festgelegt: a := „Ich bestehe die Klausur“ und b := „Ich freue mich“.
Formulieren Sie nachfolgende Aussagen in aussagenlogische Formeln ausschließlich mittels der drei Junktoren ¬, ∧ und ∨, so dass die Reihenfolge
der aussagenlogischen Variablen soweit möglich erhalten bleibt:
(a) „weder bestehe ich die Klausur noch freue ich mich“
(b) „auf jeden Fall bestehe ich die Klausur unabhängig davon, ob ich mich
freue“
(c) „dann bestehe ich die Klausur, wenn ich mich freue“
(d) „auf keinen Fall freue ich mich“
(e) „entweder bestehe ich die Klausur oder ich freue mich“
(f ) „Ich kann nicht zugleich die Klausur bestehen und mich freuen“
(g) „Ich werde mindestens eins von beidem tun: die Klausur bestehen oder
mich freuen“
Beispiel: „Ich bestehe die Klausur und ich freue mich“: a ∧ b.
21. Modellierung (? Punkte)
Hawkeye hat beim Baseball seinen Golfschläger zertrümmert und braucht
dringend einen neuen. Er wendet sich mit seinem Problem an Radar. Der
schwingt sich ans Funkgerät und legt nach einiger Zeit die folgende Liste
mit möglichen Tauschgeschäften vor:
• Morphium + reiner Alkohol → 1 Schinken + 1 Flasche Whisky
• 1 Flasche Whisky + 1 Flasche Selbstgebrannter → 1 Lenkrad
• 1 Tüte Gips + 1 Schinken → 1 Topf + 1 Grammophon
• 1 Grammophon + 1 Lenkrad → 1 Golfschläger
Im Lager vorhanden sind Morphium, reiner Alkohol, Selbstgebrannter und
Gips. Bekommt Hawkeye seinen Golfschläger ?
(a) Wandeln Sie diese Fragestellung in eine Erfüllbarkeitsfrage für eine
geeignete aussagenlogische Formel um.
(b) Lösen Sie die Erfüllbarkeitsfrage für diese Formel mit Hilfe des Resolutionskalkül.
A.1. ÜBUNGEN LOGIK
15
22. Modellierung (? Punkte)
Bauer Horst besitzt einen kleinen Acker, einen Wolf und das Schwein Borsti. Wenn der Händler ehrlich ist, ist das Saatgut, das Horst von ihm kauft,
gut. Wenn das Saatgut und das Wetter im Sommer gut sind, verdient Horst
ausreichend Geld. Wenn er ausreichend Geld hat, füttert er den Wolf. Wenn
er den Wolf gefüttert hat und in der Stadt Kirmes ist, geht er in die Stadt
und lässt Wolf und Borsti allein. Wenn der Wolf gefüttert worden ist, ist er
satt. Wenn der Wolf satt ist oder der Bauer anwesend ist, wird Borsti nicht
gefressen.
(a) Wandeln Sie diesen Text in eine Erfüllbarkeitsfrage für eine geeignete
aussagenlogische Formel in KNF um.
(b) Angenommen, in der Stadt ist Kirmes. Wird Borsti nicht gefressen,
wenn das Wetter im Sommer gut und der Händler ehrlich waren ?
Lösen Sie diese Erfüllbarkeitsfrage für die Formel aus Teil (a) mit
Hilfe des Resolutionskalkül.
23. Modellierung (? Punkte)
Geben Sie eine Formel F an, die drei atomare Formeln A, B und C enthält und folgende Eigenschaften erfüllt: Für jede Belegung A : {A, B, C} →
{0, 1} gilt, dass das Ändern irgendeiner der Werte A(A), A(B) oder A(C)
auch A(F ) ändert. Beispiel: Sei o.B.d.A. F (1, 0, 1) = 1, dann müsste
F (0, 0, 1) = F (1, 1, 1) = F (1, 0, 0) = 0 sein.
Tipp: Stellen Sie zunächst eine Formel auf, die nur zwei atomare Formeln
beinhaltet und die Änderung der Eingabe signalisiert.
24. Modellierung (? Punkte)
Nach dem ersten Semester an der HAW Hamburg fühlen Sie sich nun fit
in Logik und können erklären, ob folgender Schluß richtig oder falsch ist:
„Wenn ich 100m unter 10 Sekunden laufe, werde ich zur Olympiade zugelassen. Da ich die 100m nicht unter 10 Sekunden laufe, werde ich folglich nicht
zur Olympiade zugelassen“. Verwenden Sie zur Erklärung die Aussagenlogik, d.h. leiten Sie mittels der Aussagenlogik die Gültigkeit oder Ungültigkeit
dieser Aussage ab!
25. Modellierung (? Punkte)
Auf die Frage, worin sein Geheimnis für sein langes Leben besteht, antwortete ein 100-jähriger wie folgt: „Ich halte mich streng an die Diätregel:
Wenn ich keinen Wein zur Mahlzeit trinke, dann habe ich immer Fisch.
Immer wenn ich Fisch und Wein zur selben Mahlzeit habe, verzichte ich auf
Schokolade. Wenn ich Schokolade habe oder Wein meide, dann rühre ich
Fisch nicht an“. Der Fragensteller fand diesen Ratschlag ziemlich verwirrend. Können Sie ihn mit Hilfe der Aussagenlogik vereinfachen ?
16
ANHANG A. ÜBUNGSAUFGABEN
26. Modellierung (? Punkte)
Zwei Logiker, beide in Rente, eröffnen jeder ein Restaurant. Sie kommen
überein, folgende Ruhetag-Regelung streng logisch durch folgende Aussagevariablen zu erklären:
Restaurant 1 „Wenn Montag ist, dann ist Ruhetag“
Restaurant 2 „Nur wenn es Montag ist, dann ist Ruhetag“
Formalisieren Sie obige Erklärungen, füllen Sie eine Wahrheitstafel aus und
interpretieren Sie deren Zeilen. Genau das ist (hier) mit „logischem Denken“ gemeint!
27. Modellierung (? Punkte)
Vorgelegt seien die Aussagenvariablen (Sätze)
a : „Wiebke ist Diplom-Ingenieurin“
b : „Wiebke ist verheiratet“
c : „Wiebke wohnt in Hamburg“
Modelieren Sie mittels den Aussagenvariablen in kanonischer Normalform
(KNF oder DNF) folgende Formalisierungen:
(a) Mindestens einer der drei Sätze gilt.
(b) Genau einer der drei Sätze gilt, d.h. einer und nur einer gilt.
(c) Höchstens einer der drei Sätze gilt.
28. Modellierung (? Punkte)
Schon kurz nach der Geburt der Zwillingsbrüder Herakles und Eurystheus
entstand ein Streit, wer von den beiden der rechtmäßige Herrscher sei. Dazu
wurden die drei bekanntesten Orakel befragt. Das Ammonion gab bekannt,
dass die Orakelsprüche aus Klaros grundsätzlich falsch seien. Ebenso ließ
das Orakel aus Klaros verlauten, dass die Orakelsprüche aus Delphi samt
und sonders unzutreffend seien. Das Orakel aus Delphi jedoch behauptete,
sowohl die Sprüche des Ammonions als auch die des Orakels in Klaros seien
unwahr.
Wem sollten die armen Griechen nun glauben? Lösen Sie die Aufgabe mittels Wahrheitstafel.
29. Modellierung (? Punkte)
Ede, Max und Otto werden des Diebstahls verdächtigt. Sie können jeweils
ein notorischer Lügner sein, d.h. nicht die Wahrheit sagen, oder ehrlich
sein.
A.1. ÜBUNGEN LOGIK
17
• Otto sagt: „Ede ist der Täter!“
• Ede sagt: „Ich bin unschuldig!“
• Max sagt: „Otto lügt!“
Die Indizien machen deutlich, dass hier genau zwei lügen oder gar keiner
lügt. Ermitteln Sie mittels Wahrheitstafel die Lösung. Wer ist jeweils der
Täter?
30. Modellierung (? Punkte)
Captain Archer und Sub-Commander T’Pol landen auf dem Planeten Wahrlüg, auf dem die Bewohner immer lügen oder immer die Wahrheit sagen.
Sie treffen auf drei Bewohner:
• Der erste sagt: „Einer von den beiden anderen ist ein Lügner.“
• Der zweite sagt: „Wenn das wahr ist, dann ist der dritte der Lügner.“
• Der dritte sagt: „Nur wenn der erste die Wahrheit gesagt hat, dann ist
der Zweite ein Lügner.“
Wen sollten die beiden als Ortskundigen mitnehmen ? Lösen Sie die Frage
mittels einer Wahrheitstafel.
31. Modellierung (?? Punkte)
Donald war erbost: „Wer von euch hat von der Torte genascht ?“ Seine
Neffen blickten betreten auf das Backwerk, dessen kunstvolle Dekoration von
kleinen Entenfingern (wie auch immer die aussehen) übel zugerichtet war.
Einerseits wollten sie nicht petzen oder sich selbst beschuldigen, andererseits
wollten sie ihren Onkel auch nicht belügen.
• Nach einigen Anläufen quälte Tick aus sich heraus: „Trick oder ich
waren es.“
• Dann druckste Trick: „Entweder war es Track oder ich.“
• Track gab an: „Entweder hat Tick oder ich nicht genascht.“
Zum Erstaunen der drei wusste Donald sofort, wer genascht hatte. Nämlich ?
Begründen Sie kurz, warum Sie sich für das von Ihnen gewählte Verfahren
zur Klärung der Frage entschieden haben.
18
ANHANG A. ÜBUNGSAUFGABEN
32. Modellierung (? Punkte)
Von der Schwierigkeit, Menschen zu verstehen:
• Anne sagt: „Bettina sagt die Wahrheit.“
• Bettina sagt: „Claudia lügt.“
• Claudia sagt: „Anne und Bettina sagen beide die Wahrheit oder lügen
beide.“
Wer lügt denn nun und wer sagt die Wahrheit ? Beantworten Sie die Frage
mittels Resolutions Kalkül und behaupten dazu, dass Claudia die Wahrheit
sagt.
33. Modellierung (? Punkte)
Übersetzen Sie folgende Texte in aussagenlogische Formeln. Bestimmen Sie
dazu zunächst die elementaren/atomaren Aussagen (im Sinne aussagenlogischer Variablen) und ordnen Sie diesen Buchstaben als Abkürzung zu;
Beispiel: K = „Ich habe Recht“.
(a) „Platon hatte Recht mit seiner Einschätzung des Sokrates genau dann,
wenn Sokrates kein großer Philosoph war.“
(b) „Wenn Sokrates ein großer Philosoph war, dann hatte Aristoteles Recht
mit seiner Einschätzung des Platon.“
(c) „Aristoteles hatte nur dann Recht mit seiner Einschätzung des Platon,
falls Platon Recht hatte mit seiner Einschätzung des Sokrates.“
(d) „Unter der Annahme, dass zu all den bisherigen Aussagen noch gilt,
dass Platons Einschätzung des Sokrates stimmt, folgt, dass Sokrates
kein großer Philosoph war.“
Hinweis: Sie können →, ↔, ∧, ∨, ¬ verwenden.
34. Modellierung (? Punkte)
Gegeben seien folgende Aussagen:
• Wenn Elmo Aktien der Logik-AG kauft, dann erhält er viel Dividende,
falls der aktuelle Direktor der Logik-AG von seinem Posten abgelöst
wird.
• Elmo wird sich kein Haus bauen, falls er viel Dividende erhält und
heiratet.
• Wenn Elmo im Sommer nicht nach Madeira fährt, dann wird er heiraten und sich ein Haus bauen.
• Falls also Elmo Aktien der Logik-AG kauft, dann fährt er im Sommer
nach Madeira, wenn der aktuelle Direktor der Logik-AG von seinem
Posten abgelöst wird.
A.1. ÜBUNGEN LOGIK
19
(a) Erstellen Sie eine aussagenlogische Formel aus den Aussagen.
(b) Zeigen Sie mittels Resolutions-Kalkül, dass die vierte Formel (letzte
bzw. unterste Aussage) aus den vorangegangenen Formeln folgt.
35. Modellierung (? Punkte)
Sie kommen auf eine Insel mit zwei Stämmen, den Wahrheitssprechern und
den Lügensprechern. Die Wahrheitssprecher sagen immer die Wahrheit, die
Lügensprecher lügen immer.
Beim Landen auf der Insel treffen Sie Alice, Bob und Charlie. Formalisieren
Sie die nachfolgenden Situation mit Hilfe einer aussagenlogischen Formel.
Wählen Sie die aussagenlogischen Variablen so, dass Sie die Situationen in
(b) und (c) auch zusammen betrachten können.
(a) Klären Sie, wie der Begriff „sagt“ zu modellieren ist, also wie wird „Jemand sagt etwas“ in einer aussagenlogischen Formel modelliert, unter
der Voraussetzung, dass Jemand sowohl Wahrheitssprecher wie auch
Lügensprecher sein kann. Verwenden Sie dazu ausschließlich die zwei
aussagenlogischen Variablen JW: „Jemand ist Wahrheitssprecher“ und
E: „Etwas“.
Begründen Sie Ihre Antwort ausführlich.
Modellieren Sie mit der gefundenen Formel als kleines Beispiel folgende Situation: Hans sagt: „Es regnet“ mittels den beiden aussagenlogischen Variablen HW für Hans ist Wahrheitssprecher und R für Es
regnet. Geben Sie für die Formel eine Wahrheitstafel an und interpretieren Sie das Ergebnis.
(b) Sie fragen Alice, zu welchem Stamm sie gehöre. Alice versteht Sie wegen Ihrem Akzent nicht und Bob dolmetscht. Bob sagt, Alice habe gesagt, sie sei eine Lügensprecherin. Charlie mischt sich ein und sagt,
Bob lüge und Alice sei eine Wahrheitsprecherin.
Können Sie entscheiden, zu welchen Stämmen Alice, Bob und Charlie
gehören? Begründen Sie Ihre Antwort ausführlich durch z.B. Berechnung / Umformung oder eine Wahrheitstafel.
(c) Sie hören das folgende Gespräch: Alice sagt: „Charlie sagt die Wahrheit.“. Bob sagt: „Wenn Alice lügt, dann sage ich die Wahrheit.“ Charlie
sagt: „Wenn Bob lügt, dann lügt auch Alice.“
Ist Charlie vom Stamme der Lügensprecher? Begründen Sie Ihre Antwort ausführlich durch z.B. Berechnung / Umformung oder eine Wahrheitstafel.
(d) Widersprechen sich die Situationen in (b) und (c) oder stimmen sie
überein? Begründen Sie auch hier Ihre Antwort!
20
ANHANG A. ÜBUNGSAUFGABEN
36. Modellierung (? Punkte)
In der Stadt Sörknatten besteht die Bevölkerung ausschließlich aus Rittern
und Schurken, wobei Ritter immer die Wahrheit sagen, und Schurken immer
lügen.
Beim Eintreffen in der Stadt Sörknatten treffen Sie Kunibert, Hotzenblotz
und Siegfried. Formalisieren Sie die nachfolgende Situation mit Hilfe einer
aussagenlogischen Formel.
(a) Klären Sie, wie der Begriff „sagt“ zu modellieren ist, also wie wird „Jemand sagt etwas“ in einer aussagenlogischen Formel modelliert, unter
der Voraussetzung, dass Jemand sowohl Ritter wie auch Schurke sein
kann. Verwenden Sie dazu ausschließlich die zwei aussagenlogischen
Variablen JR: „Jemand ist Ritter“ und E: „Etwas“.
Begründen Sie Ihre Antwort ausführlich.
Modellieren Sie mit der gefundenen Formel als kleines Beispiel folgende Situation: Kunibert sagt: „Es regnet“ mittels den beiden aussagenlogischen Variablen KR für Kunibert ist Ritter und R für Es regnet.
Geben Sie für die Formel eine Wahrheitstafel an und interpretieren
Sie das Ergebnis.
(b) Sie fragen alle Drei, ob sie Ritter oder Schurken seien. Kunibert sagt:
„Einer von den beiden anderen ist ein Schurke.“ Hotzenblotz sagt:
„Wenn das wahr ist, dann ist Siegfried der Schurke.“ Siegfried sagt:
„Hotzenblotz ist nur ein Schurke, wenn Kunibert die Wahrheit gesagt
hat.“
Können Sie entscheiden, wer von den Dreien Ritter oder Schurke ist?
Begründen Sie Ihre Antwort ausführlich durch z.B. Berechnung / Umformung oder eine Wahrheitstafel.
37. Modellierung (? Punkte)
Meiko möchte seinen Geburtstag feiern. Leider sind seine Freunde Emmanuela, Samuel, Ronja und Neo recht schwierig. Und zwar ist es so, dass
• Emmanuela kommt nur, wenn auch Samuel kommt.
• Samuel kommt nur, wenn Ronja kommt.
• Wenn wiederum Ronja kommt, kommt auch Neo.
• Wenn allerdings Samuel und Neo kommen, kommt Ronja nicht.
• Neo kommt nur, wenn Emmauela oder Samuel kommen.
(a) Stellen Sie eine aussagenlogische Formel auf, die die obige Situation
beschreibt.
(b) Zeigen Sie, dass keiner dieser vier Freunde zu Meikos Geburtstagsfeier
kommt.
A.1. ÜBUNGEN LOGIK
21
38. Modellierung (? Punkte)
Modellieren Sie die nachfolgenden Argumentationen und beantworten Sie
die Fragen mittels Resolution. Gegeben seien dazu die nachfolgenden Voraussetzungen:
(a) Peter bekommt in der Prüfung eine gute Note genau dann, wenn er
lernt, alle Vorlesungen besucht und genügend schläft.
(b) Peter kann nicht genügend schlafen, wenn er ständig in die Disko geht.
(c) Peter besucht alle Vorlesungen, lernt, geht ständig in die Disko und
wird dort von seinem Professor gesehen.
Erste Frage: Bekommt Peter in der Prüfung eine gute Note ?
Zweite Frage: Zu welchem Ergebnis kommen Sie, wenn die erste Prämisse
durch nachfolgende ersetzt wird ?
(a’) Wenn Peter lernt oder alle Vorlesungen besucht oder genügend schläft,
so bekommt er in der Prüfung eine gute Note.
39. Modellierung (? Punkte)
Anja, Bernd und Claudia wollen nach Schweden fahren. Da dort insbesondere Alkohol sehr teuer ist, wollen sie Aquavit, Bier und Cognac mitnehmen.
Wie jeder weiß, gehört zu jedem Gläschen Aquavit ein Bierchen, vertragen
sich Bier und Cognac nicht, und ist Cognac nicht gern allein im Magen.
Man zeige, dass sich die drei die Flasche Cognac sparen können, indem
man
(a) eine Wahrheitstafel aufstellt.
(b) mit Hilfe fundamentaler Äquivalenzen zeigt, dass obige Aussagen zusammengefasst werden können zu der Formel „Wenn Aquavit, dann
auch Bier, und niemals Cognac.“
40. Modellierung (? Punkte)
Vor dem letzten Geburtstag des Bären planten der Hamster und der Wolf
einen Kuchen für den Bären zu backen. Über das Rezept gerieten Sie jedoch
in Streit. Der Wolf sagte: „Wenn Eier in den Kuchen kommen, dann isst der
Bär nichts von dem Kuchen!“. Der Hamster widersprach: „Fehlen die Eier,
dann kann man keinen Apfelkuchen backen! Und der Bär isst genau dann
Kuchen, wenn es Apfelkuchen gibt.“ „Aber der Bär muss von dem Kuchen
essen!“ entgegnete der Wolf.
(a) Stellen Sie eine aussagenlogische Formel auf, die die obige Situation
beschreibt.
22
ANHANG A. ÜBUNGSAUFGABEN
(b) Prüfen Sie mittels Resolutions-Kalkül, ob dies ein korrektes Ende der
Geschichte sein könnte: Da rief der Hamster: „Das geht doch gar
nicht!“ − „Wir müssen das aber hin bekommen!“ brüllte der Wolf. Der
Wolf verbat dem Hamster darauf hin das Durchqueren seines Waldes
und versuchte erfolglos den Kuchen zu backen.
41. Modellierung (? Punkte)
Ein Bäcker möchte Rosinenbrötchen backen. Leider hat er nicht alle nötigen
Zutaten (ihm fehlen die Rosinen), er kann jedoch einige vorhandene Zutaten
gegen andere tauschen, und zwar:
• Mehl + Eier → Milch + Honig
• Mandeln + Honig → Rosinen
• Milch + Hefe → Mandeln
• In großer Menge vorhanden sind Mehl, Eier und Hefe.
(a) Übersetzen Sie die Aussagen in aussagenlogische Formeln und formulieren Sie die Frage, ob der Bäcker Rosinen erhält, als Unerfüllbarkeitsproblem.
42. Modellierung (? Punkte)
Der Fakultätsrat eines norddeutschen Informatikdepartements beschloss eine neue Prüfungsordnung für einen Bachelorstudiengang. Es werden Scheine für die Fächer Theologik, Praxologie und Schaltkreislöten sowie für ein
Grünkohlpraktikum vergeben. Für den Bachelorabschluss müssen alle folgenden Bedingungen erfüllt werden:
(a) Wurde ein Grünkohlschein nicht erworben, so muss ein Praxologieschein
vorhanden sein.
(b) Fehlt der Praxologieschein oder fehlt der Schaltkreisschein, so müssen
der Theologikschein und der Grünkohlschein vorhanden sein.
(c) Wurden weder der Theologikschein noch der Praxologieschein erworben, so müssen der Schaltkreisschein und der Grünkohlschein vorhanden sein.
Das zuständige Kultusministerium lehnte die Prüfungsordnung wegen „undurchsichtiger Formulierungen“ ab. Das Departement wurde aufgefordert,
die Scheinerwerbsbedingungen so umzuformulieren, dass möglichst wenige,
einfache Alternativen entstehen. Helfen Sie dem Dekan bei dieser schweren
Arbeit!
A.1. ÜBUNGEN LOGIK
23
43. Modellierung (? Punkte)
Nachdem Ensel und Krete die Hexe in den Ofen gestoßen haben, wollen sie
sich nun über das Knusperhäuschen hermachen. Aber wie allgemein bekannt
ist, muss man beim Verspeisen eines solchen Hauses sehr vorsichtig sein,
da diese Häuser zur Instabilität neigen. Die beiden wenden sich zunächst
einer Wand zu, die aus drei Lebkuchen besteht. Da Ensel erfolgreich Knusperhäuschenarchitektur studiert hat, erkennt er, dass folgende drei Regeln
aus Sicherheitsgründen unbedingt einzuhalten sind:
• Von den beiden ersten Lebkuchen darf höchstens einer entfernt werden.
• Wenn man den dritten entfernt, muss man auch den zweiten entfernen.
• Wenn man den zweiten entfernt und den ersten nicht, dann darf man
den dritten nicht entfernen.
Da Krete in Logik aufgepasst hat, weiss sie, dass man vom dritten Lebkuchen
besser die Finger lässt.
(a) Stellen Sie für die beschriebenen Regeln eine aussagenlogische Formel
auf.
(b) Prüfen Sie mittels Resolutions-Kalkül, ob Krete Recht hat.
(c) Prüfen Sie mittels Wahrheitstafel, ob Krete Recht hat.
44. Äquivalenz und Implikation (? Punkte)
In einer Theorieklausur wird von den Studierenden der Nachweis der Nullteilerfreiheit in Körperstrukturen verlangt. Unter Beschränkung auf die Aussagenlogik soll also die Gültigkeit des Ausdrucks
(1)(a ∧ b = 0) → (a = 0 ∨ b = 0)
(für alle a,b des Körpers) gezeigt werden. Diese Aufgabe steht hier nicht
an!! Sondern: Bei der Korrektur stellt es sich heraus, dass eine Reihe von
Studenten nicht (1), sondern andere Aussagen bewiesen haben, nämlich
(2)((a 6= 0) ∧ (b = 0)) → (a ∧ b = 0)
(3)((a 6= 0) ∧ (b 6= 0)) → (a ∧ b 6= 0)
(4)(a ∧ b = 0) ↔ (a = 0 ∨ b = 0)
Gefragt wird, mit welchen der Formeln (2) bis (4) zugleich – und zwar aus
logischen Gründen – auch (1) bewiesen worden ist. Helfen Sie dem Professor
bei der Korrektur, indem Sie die Äquivalenz bzw. Implikation der Ausdrücke
(2) bis (4) bezüglich (1) zeigen.
Tipp: Formalisieren Sie gemäß A := (a = 0); B := (b = 0); C := ((a ∧
b) = 0) und überlegen Sie den Nutzen von (konjunktiver oder disjunktiver)
Normalform!
24
ANHANG A. ÜBUNGSAUFGABEN
45. neue Junktoren (? Punkte)
Zeigen Sie, dass die Junktorenmenge {◦, ⋆} vollständig für die Aussagenlogik ist. Das heißt, man kann jeden Junktor der Aussagenlogik mit Hilfe
dieser beiden Junktoren darstellen. Dabei sind ◦ (Replikation: „nur wenn ...,
dann ...“) und ⋆ ((Postsektion: „das eine ohne das andere“) folgendermaßen
definiert:
a ⋆ b := a ∧ ¬b und a ◦ b := a ∨ ¬b
Definieren Sie ¬, ∧ und ∨ ausschließlich mittels den beiden neuen Junktoren
◦ und ⋆ und weisen Sie die Korrektheit mittels einer Wahrheitstafel oder
durch direkte Umformung nach. Beginnen Sie mit dem Junktor ¬.
46. neue Junktoren (? Punkte)
Zeigen Sie jeweils, dass der Junktor | (Shefferscher Strich: „nicht sowohl ...
als auch ...“) sowie der Junktor ⇓ (Peirce-Pfeil: „weder ... noch ...“) jeweils
vollständig für die Aussagenlogik sind. Das heißt, man kann jeden Junktor
der Aussagenlogik mit Hilfe einer dieser beiden Junktoren darstellen. Dabei
sind | und ⇓ folgendermaßen definiert:
a|b := ¬(a ∧ b) und a ⇓ b := (¬a) ∧ (¬b)
Definieren Sie ¬, ∧ und ∨ ausschließlich mittels einem dieser beiden neuen
Junktoren | und ⇓ und weisen Sie die Korrektheit mittels einer Wahrheitstafel oder durch direkte Umformung nach. Beginnen Sie mit dem Junktor
¬.
47. neue Junktoren (? Punkte)
Zeigen Sie, dass die Junktorenmenge {→, 6←} vollständig für die Aussagenlogik ist. Das heißt, man kann jeden Junktor der Aussagenlogik mit Hilfe
dieser beiden Junktoren darstellen. Dabei sind → (Implikation: „wenn ...
dann ...“) und 6← (Präsektion: „das andere ohne das eine“) folgendermaßen
definiert:
a → b := ¬a ∨ b und a ≤ b := ¬a ∧ b
Definieren Sie ¬, ∧ und ∨ ausschließlich mittels den beiden neuen Junktoren
→ und ≤ und weisen Sie die Korrektheit mittels einer Wahrheitstafel oder
durch direkte Umformung nach. Beginnen Sie mit dem Junktor ¬.
48. Basis (? Punkte)
Weisen Sie die Korrektheit mittels einer Wahrheitstafel oder durch direkte
Umformung mittels den fundamentalen Äquivalenzen nach.
(a) Zeigen Sie, dass die Junktorenmenge {∧, ¬} vollständig für die Aussagenlogik ist.
(b) Zeigen Sie, dass die Junktorenmenge {∨, ¬} vollständig für die Aussagenlogik ist.
A.1. ÜBUNGEN LOGIK
25
(c) Zeigen Sie, dass die Junktorenmenge {→, ¬} vollständig für die Aussagenlogik ist.
Als bekannt vorausgesetzt ist, dass die Junktorenmenge {∧, ∨, ¬} vollständig
für die Aussagenlogik ist, also eine Basis bildet: man kann alle aussagenlogischen Formeln ausschließlich mit Hilfe dieser Junktoren darstellen.
49. KNF-Algorithmus (? Punkte)
Erzeugen Sie durch Anwendung des KNF-Algorithmus (siehe Tipp) aus
nachfolgender Formel eine KNF (Die Anwendung des Algorithmus muss
aus der Dokumentation nachvollziehbar sein!):
F = (((¬(¬B)) ∨ A) ∧ ((¬A ∧ ¬(B ∧ ¬C)) ∨ (A ∧ (B ∧ ¬C))))
Wenden Sie auf die so erzeugte KNF die Tautologieregel und die Unerfüllbarkeitsregel sowie die Distributivregel an, um den Ausdruck zu vereinfachen. Jede Anwendung einer Äquivalenzregel ist deutlich zu markieren! Als
Endergebnis ist eine KNF zu erstellen, die maximal drei Disjunktionsglieder
hat ((?1 ∨?2 ∨ . . .) ∧ (?1 ∨?2 ∨ . . .) ∧ (?1 ∨?2 ∨ . . .)).
Tipp:
Algorithmus A.1 (KNF) Gegeben sei eine Formel F. Führe die folgenden Schritte durch:
(a) Ersetze in F jede Teilformel der Form ¬¬G durch G.
(b) Ersetze in F jede Teilformel der Form ¬(G ∧ H) durch (¬G ∨ ¬H).
Entsteht hierdurch eine Teilformel der Form ¬¬K, so wende Schritt
(a) an.
(c) Ersetze in F jede Teilformel der Form ¬(G ∨ H) durch (¬G ∧ ¬H).
Entsteht hierdurch eine Teilformel der Form ¬¬K, so wende Schritt
(a) an.
(d) Wiederhole die Schritte (b) und (c) so oft wie möglich.
(e) Ersetze in F jede Teilformel der Form (G ∨ (H ∧ I)) durch ((G ∨ H) ∧
(G ∨ I)).
(f ) Ersetze in F jede Teilformel der Form ((G ∧ H) ∨ I)) durch ((G ∨ I) ∧
(H ∨ I)).
(g) Wiederhole die Schritte (e) und (f ) so oft wie möglich.
26
ANHANG A. ÜBUNGSAUFGABEN
Benötigte fundamentale Äquivalenzen sind:
D (F ∧ (G ∨ H)) ≡ ((F ∧ G) ∨ (F ∧ H)) sowie
(F ∨ (G ∧ H)) ≡ ((F ∨ G) ∧ (F ∨ H))
T Ist F eine Tautologie, so gilt (F ∨ G) ≡ F und (F ∧ G) ≡ G.
U Ist F unerfüllbar, so gilt (F ∨ G) ≡ G und (F ∧ G) ≡ F .
50. KNF-Algorithmus (? Punkte)
Erzeugen Sie ausschliesslich durch Anwendung des KNF-Algorithmus
(siehe Tipp) aus nachfolgender Formel eine KNF (Die Anwendung des Algorithmus muss aus der Dokumentation nachvollziehbar sein!):
F = ¬(([((P ↔ ¬S) ∧ (S → A)) ∧ (A → P )] ∧ P ) → ¬S)
51. DNF-Algorithmus (? Punkte)
Erzeugen Sie durch Anwendung des DNF-Algorithmus (siehe Tipp) aus
nachfolgender Formel eine DNF. Der DNF-Algorithmus arbeitet analog zu
dem KNF-Algorithmus, erzeugt jedoch eine disjunktive Normalform und
keine konjunktive Normalform! (Die Anwendung des Algorithmus muss aus
der Dokumentation nachvollziehbar sein!):
F = (((¬(¬B)) ∨ A) ∧ ((¬A ∧ ¬(B ∧ ¬C)) ∨ (A ∧ (B ∧ ¬C))))
Wenden Sie auf die so erzeugte DNF die Idempotenzregel, Tautologieregel,
Unerfüllbarkeitsregel sowie die Distributivregel an, um den Ausdruck zu vereinfachen. Jede Anwendung einer Äquivalenzregel ist deutlich zu markieren!
Als Endergebnis ist eine DNF zu erstellen, die maximal zwei Konjunktionsglieder hat ((?1 ∧?2 ∧ . . .) ∨ (?1 ∧?2 ∧ . . .)).
Tipp:
Algorithmus A.2 (DNF) Gegeben sei eine Formel F. Führe die folgenden Schritte durch:
(a) Ersetze in F jede Teilformel der Form ¬¬G durch G.
(b) Ersetze in F jede Teilformel der Form ¬(G ∧ H) durch (¬G ∨ ¬H).
Entsteht hierdurch eine Teilformel der Form ¬¬K, so wende Schritt
(a) an.
(c) Ersetze in F jede Teilformel der Form ¬(G ∨ H) durch (¬G ∧ ¬H).
Entsteht hierdurch eine Teilformel der Form ¬¬K, so wende Schritt
(a) an.
(d) Wiederhole die Schritte (b) und (c) so oft wie möglich.
A.1. ÜBUNGEN LOGIK
27
(e) Ersetze in F jede Teilformel der Form (G ∧ (H ∨ I)) durch ((G ∧ H) ∨
(G ∧ I)).
(f ) Ersetze in F jede Teilformel der Form ((G ∨ H) ∧ I) durch ((G ∧ I) ∨
(H ∧ I)).
(g) Wiederhole die Schritte (e) und (f ) so oft wie möglich.
Benötigte fundamentale Äquivalenzen sind:
I (F ∧ F ) ≡ F sowie (F ∨ F ) ≡ F
D (F ∧ (G ∨ H)) ≡ ((F ∧ G) ∨ (F ∧ H)) sowie
(F ∨ (G ∧ H)) ≡ ((F ∨ G) ∧ (F ∨ H))
T Ist F eine Tautologie, so gilt (F ∨ G) ≡ F und (F ∧ G) ≡ G.
U Ist F unerfüllbar, so gilt (F ∨ G) ≡ G und (F ∧ G) ≡ F .
52. Resolutions-Kalkül (? Punkte)
Geben Sie sämtliche Resolventen an, die aus den Klauseln der Klauselmenge
{{A, ¬B, E}, {A, B, C}, {¬A, ¬D, E}, {A, ¬C}}
gewonnen werden können.
53. Resolutions-Kalkül (? Punkte)
Kann bei der Resolution zweier Hornformeln eine Formel entstehen, die
keine Hornformel ist ?
54. Resolutions-Kalkül (? Punkte)
Zeigen Sie mittels Resolutionsmethode, dass
F = ((¬A ∨ B) ∧ (¬B ∨ C) ∧ (A ∨ ¬C) ∧ (A ∨ B ∨ C) ∧ (¬A ∨ ¬B ∨ ¬C))
unerfüllbar ist, d.h. die leere Klausel hergeleitet werden kann.
55. Resolutions-Kalkül (? Punkte)
Wenden Sie auf die nachfolgende Formel das Resolutions-Kalkül (Algorithmus: siehe im Tipp) wie folgt an:
(a) Erzeugen Sie mittels den Klauseln alle direkten Resolventen.
(b) Erzeugen Sie alle Resolventen, die mittels Resolution aus der zu (¬b →
c) gehörenden Klausel und jeweils den (unter (a), wie auch hier neu)
erzeugten Resolventen entstehen, bis keine neuen Resolventen gebildet
werden können.
Transformieren Sie dazu nachvollziehbar die Formel in eine KNF (benötigte
Äquivalenzen und Definitionen: siehe Tipp)
F = (¬b → c) ∧ ((¬c → b) → a) ∧ (¬b → (a → ¬c))
28
ANHANG A. ÜBUNGSAUFGABEN
56. Resolutions-Kalkül (? Punkte)
Testen Sie mittels Resolution folgende KNF (ohne Vereinfachungen vorzunehmen!) auf Widersprüchlichkeit:
F = P ∧(¬P ∨A)∧(P ∨¬P )∧(P ∨S)∧S ∧(¬S ∨A)∧(¬S ∨¬P )∧(¬S ∨S)
Transformieren Sie dazu die Formel in die Mengenschreibweise. Identifizieren Sie die Klauseln und führen dann den Algorithmus aus.
57. Resolutions-Kalkül (? Punkte)
Folgende vier Formeln seien gegeben:
P1 = (a ∧ b) → ¬c P2 = (b ∨ c) → a P3 = (a ∧ ¬b) → ¬c P4 = ¬a → c
Zeigen Sie mittels Resolutions-Kalkül, ob folgende Formeln eine Folgerung
aus P = P1 ∧ P2 ∧ P3 ∧ P4 sind:
(a) F1 = a
(b) F2 = (a ∧ ¬c)
(c) Wenn P5 = ¬(b → b) zu P hinzugefügt wird, welchen Einfluss hat dies
auf die Folgerung einer beliebigen Formel F aus P ∧ P5 ?
58. Resolutions-Kalkül (? Punkte)
Gegeben seien folgende Aussagen:
In einer Firma wird Team 1 die Projekte A und B vollenden,
wenn die Statistik C vorliegt. Team 2 wird Projekt D vollenden, wenn Projekt A fertig ist und die Statistik E vorliegt.
Team 3 wird Projekt F fertig stellen, wenn die Projekte B
und D abgeschlossen sind.
(a) Übersetzen Sie diesen Text in eine aussagenlogische Hornformel (Nur
eine Schlußfolgerung bei →!). Bestimmen Sie dazu zunächst die elementaren/atomaren Aussagen (im Sinne aussagenlogischer Variablen)
und ordnen Sie diesen Buchstaben als Abkürzung zu; Beispiel: K =
„Ich habe Recht“.
Hinweis: Verwenden Sie die Symbole →, ∧, ∨, ¬.
(b) Erzeugen Sie ausschließlich durch Anwendung des KNF-Algorithmus
(siehe Tipp) aus der in (a) gewonnenen Formel eine KNF (Die Anwendung des Algorithmus muss aus der Dokumentation nachvollziehbar sein!).
(c) Kann Projekt F fertig gestellt werden, wenn die Statistiken C und E
vorliegen ?
A.1. ÜBUNGEN LOGIK
29
Formulieren Sie diese Frage als eine aussagenlogische Formel, negieren
Sie diese Formel und transformieren Sie diese (gemäß dem Algorithmus) in eine KNF. Verknüpfen Sie nun diese KNF mittels ∧ mit der in
(b) erhaltenen KNF zu einer neuen KNF und testen Sie diese Formel
mittels dem Resolutionskalkül (siehe Tipp) auf Widersprüchlichkeit.
59. Resolutions-Kalkül (? Punkte)
Zeigen Sie mittels Resolutions-Kalkül, dass nachfolgende Formel eine Tautologie ist:
(A ∧ (¬B → ¬A)) → B
Zeigen Sie zusätzlich mittels Wahrheitstafel, dass diese Formel eine Tautologie ist.
60. Resolutions-Kalkül (? Punkte)
(a) Zwei Formeln F und G schließen einander aus, wenn es keine Interpretation gibt, welche beide Formeln erfüllt. Wie kann man mit dem
Resolutions-Kalkül prüfen, ob zwei Formeln F und G einander ausschließen ?
(b) Zeigen bzw. begründen Sie, dass sich die Erfüllbarkeit einer Klauselmenge K nicht ändert, wenn man aus K alle Klauseln ausschließt bzw.
löscht, die eine Variable enthalten, welche nur positiv oder nur negativ
in K vorkommt.
(c) Verwenden Sie die Methoden aus Teil (a) und (b), um zu zeigen, dass
F := (X → (Y ∨Z))∧(Z → V )∧(Y ∨V )∧(Z → (¬X∨V ))∧(V ∨¬Y ∨¬Z)
und
G := ((Z ∧¬V ) → X)∧¬(¬V ∨¬Z)∧(X → (V ∨Z))∧(V ∨¬X ∨¬Y )∧
((¬V ∧ ¬X) → (Y ∨ Z)) ∧ ¬(¬V ∧ X)
einander nicht ausschließen.
61. Modellierung (?? Punkte)
Eine Person macht die folgenden Aussagen:
• Wenn ich genug Geld gespart habe, kaufe ich mir ein Auto.
• Wenn ich nicht genug Geld gespart habe, kaufe ich mir ein Fahrrad.
• Wenn ich ein Auto kaufe, fahre ich nach Spanien in den Urlaub.
• Wenn ich ein Fahrrad kaufe, bleibe ich im Urlaub daheim.
• Ich habe im Vorjahr gespartes Geld verbraucht.
30
ANHANG A. ÜBUNGSAUFGABEN
• In diesem Jahr habe ich andere Ausgaben.
• Es stimmt nicht, dass ich nicht genug Geld gespart habe und im Vorjahr gespartes Geld verbraucht habe und in diesem Jahr andere Ausgaben habe.
Fährt diese Person nach Spanien in den Urlaub?
(a) Modellieren Sie die Aussagen als aussagenlogische Formel und transformieren Sie diese in eine KNF.
(b) Stellt die resultierende KNF in eine Hornformel dar ? Wenn nicht,
welche Klauseln sind keine Horn-Klauseln ?
Würde Ihnen es gelingen, die Formel in eine Hornformel zu transformieren, wenn Sie für bestimmte aussagenlogische Variablen, z.B. die
e :≡ ¬L (und ¬L
e :≡ L) vornehVariable L, eine Ersetzung der Art L
men würden ?
(c) Lösen Sie die Fragestellung mittels dem Resolutions-Kalkül.
62. Modellierung (? Punkte)
Für die Informatikstudierenden Anton, Berta, Charlie und Dora steht demnächst die Klausur „Mathematische Grundlagen“ an. Eigentlich wollen sie
die ins nächste Semester verschieben. Sie beschließen aber, dass mindestens
einer von ihnen an dieser Klausur teilnehmen sollte, damit sie wissen, was
drankommt. Charlie weigert sich grundsätzlich, zu dieser Klausur zu gehen.
Da Anton und Dora auf ihre gemeinsame kleine Schwester aufpassen müssen, kann höchstens einer von beiden zur Klausur gehen. Dora geht nur zur
Klausur, wenn auch ihre beste Freundin Esmeralda hingeht. Wenn Esmeralda an der Klausur teilnimmt, dann auch Anton.
(a) Stellen Sie eine aussagenlogische Formel auf, die die obige Situation
bzgl. Teilnahme an dieser Klausur beschreibt.
(b) Angenommen, Berta wird krank und kann an dieser Klausur nicht
teilnehmen. Zeigen Sie mittels Resolutions-Kalkül, dass dann Anton
zur Klausur geht und Dora nicht.
63. Modellierung (? Punkte)
Inspektor Columbo hat einen Fall zu lösen. Er hat drei Personen in Verdacht, die Tat (eventuell gemeinschaftlich) begangen zu haben. Als Täter
kommen nur Martin,Franz oder Stefan in Frage. Inspektor Columbo hat die
folgenden Informationen:
(a) Wenn Martin schuldig und Franz unschuldig ist, so ist Stefan schuldig.
(b) Stefan arbeitet niemals allein.
(c) Martin arbeitet niemals mit Stefan.
A.1. ÜBUNGEN LOGIK
31
(d) Nur Martin, Franz oder Stefan kommen als Täter in Frage.
Formulieren Sie diese Sachverhalte als aussagenlogische Formeln und zeigen
Sie mittels Resolution, dass Franz ein Täter ist.
Tipp: Wie jemand arbeitet ist so direkt nicht interessant. Zur Lösung ist
hier mehr gefragt, welche Folgerung dies auf die Täterschaft hat! Also: ausnahmsweise vor der Modellierung etwas nachdenken!
64. Modellierung (? Punkte)
Wenn keine Klausur geschrieben wird, sind die Studenten glücklich. Wenn
die Studenten glücklich sind, fühlt sich der Dozent wohl. Wenn sich aber
der Dozent wohl fühlt, dann hat er keine Lust, Vorlesung zu halten. Wird
aber keine Klausur geschrieben, dann hat er Lust, Vorlesung zu halten. Also
wird eine Klausur geschrieben.
(a) Formalisieren Sie den vorliegenden Text als aussagenlogische Formel.
(b) Untersuchen Sie, ob ein korrekter Schluß vorliegt.
65. Modellierung (? Punkte)
Schneewittchen möchte in die Stadt einkaufen gehen und die Zwerge beschließen, dass mindestens einer von ihnen sie begleitet. Drei der Zwerge
scheiden als Begleitung aus, da sie Holz hacken müssen. Die möglichen Begleiter, die Zwerge Alberich, Gimli, Rübezahl und Zwergnase, diskutieren,
wer mit in die Stadt geht.
• Alberich: “Wenn Gimli nicht geht und Rübezahl nicht geht, dann geh’
ich auch nicht.“
• Gimli: „Ich gehe nur, wenn Rübezahl oder Zwergnase mitgehen.“
• Rübezahl: „Wenn Gimli nicht geht, dann gehe ich auch nicht.“
• Zwergnase: „Ich gehe nur, wenn Gimli nicht mitkommt. Wenn Rübezahl geht, dann will ich auf jeden Fall mit.“
Zeigen Sie mit Hilfe des Resolutions-Kalkül, dass die vier Zwerge nur zufrieden sind, wenn Zwergnase als einziger mit Schneewittchen in die Stadt
geht.
32
ANHANG A. ÜBUNGSAUFGABEN
A.2
Übungen Grundlagen
Aufgabe A.3 (Mengen)
1. Gegeben seien die Mengen A = {a, b}; B =
{b, c, d}; C = {b, d}; D = {a, A, B, b, C}. Welche der folgenden Beziehungen ist richtig:
a) b ∈ B; b) A ⊆ B; c) A ⊆ D; d) d ∈ D; e) C ∈ B; f ) {d, b} ⊆ D; g)
{C} ⊆ D;
h) B ∈ D; i) ∅ ⊆ A; j) ∅ ∈ B ; k) a ⊆ D ; l) B ∈ B
2. Welche der folgenden vier Aussagen ist richtig (kurze Begründung angeben!):
a) 1 ∈ {1}; b) {1} ∈ {1}; c) {1} ∈ 1; d) 1 ∈ 1; e) 1 ∈ {{1}};
f ) {1} ∈ {{1}};
3. Beschreibung (2 Punkte)
Beschreiben Sie die folgenden Mengen per expliziter Aufzählung:
A := {n|n ist Primzahl und 2 < n < 10}
B := {n|10 − n ist Primzahl und 2 < n < 10}
C := {n|∃x ∈ Z : x3 − 4x = 0 mit n = x + 5}
4. Beschreibung Beschreiben Sie die folgenden Mengen durch die Angabe
wenigstens eines Auswahlkriteriums (Prädikates) für x:
a) {1, 4, 9, 16, 25, 36, 49, 64, 81} b) {−1, 1} c) {2, 11, 101, 1001, 10001}
5. Beschreibung Gegeben sei Grundmenge G := Z die Menge der ganzen
Zahlen und es sei: A := {x ist eine Zahl, die kleiner als Null ist}
B := {x ist eine gerade Zahl}
C := {x ist eine dreistellige Zahl, deren letzte beiden Ziffern der Dezimaldarstellung 0 sind }
Geben Sie für jede der angegebenen Mengen A, B, C eine Darstellung an,
die ganz auf Wörter der deutschen Sprache bzw. einer natürlichen Sprache
verzichtet. Verwenden Sie stattdessen für endliche Mengen eine explizite
Aufzählung der Elemente und für unendliche Mengen eine formale Darstellung mit logischen und arithmetische Operatoren.
6. Beschreibung (? Punkte)
Geben Sie die folgenden Mengen in aufzählenden oder formal beschreibenden
Verfahren, d.h. mit ∈, ∃, ∀, . . . oder auch <, >, ≤, . . ., an. Als Grundmengen
können Sie P die Menge der Primzahlen und N0 die Menge der natürlichen
Zahlen (mit 0) verwenden.
(a) die Teilmenge A ⊆ N0 , deren Elemente durch 3 teilbar sind, grösser
als 10 sind und kleiner als 25.
A.2. ÜBUNGEN GRUNDLAGEN
33
(b) die Teilmenge B ⊆ N0 , deren Elemente echte Vielfache (d.h. nicht 4)
von 4 und kleiner oder gleich 96 sind.
(c) die Teilmenge C ⊆ P deren Elemente zwischen 1 und 30 sind und welche mindestens einen Primzahlzwilling in P haben (Primzahlzwillinge
sind Primzahlen p1 , p2 ∈ P mit p1 − p2 = 2).
7. Mengen (? Punkte)
Gegeben seien folgende Mengen: P = Menge der Primzahlen2 , M1 = {2, 3, 5}
und M2 = {x ∈ P |x < 10} sowie M3 = {1, 5, 7, 13, 21}.
(a) Geben Sie die Menge M2 in aufzählender Form an.
(b) Beschreiben Sie mit Mengensymbolen die Beziehung(-en) der beiden
Mengen M1 und M 2 .
(c) Bestimmen Sie die resultierenden Mengen: M1 \ M2 , (M2 ∪ M3 ) ∪ M1 ,
(M3 ∪ M2 ) ∩ M1 , (M1 ∪ M2 ) ∩ M3
8. Mengen (? Punkte)
Seien V2 , V3 und V4 die Mengen der ganzzahligen Vielfachen von 2, 3 bzw.
4 in der Grundmenge MG = {1, 2, . . . , 12}. Bestimmen Sie die Mengen
(a) R1 = V3 ∪ V4 .
(b) R2 = V4 \ V2 .
(c) R3 = (V2 ∪ V3 )−1 .
(d) R4 = V2 ∩ V3 .
(e) R5 = P (V2 ∩ V4 ).
9. Bestimmen bzw. berechnen Sie nachfolgende Ausdrücke (Hinweis: das Ergebnis der Berechnung können Mengen oder Wahrheitswerte sein!; P (M )
beschreibt die Potenzmenge von M ):
a) {} ⊆ {0, 1, 2}; b) {2} ⊆ {{0}, {1}, {2}}; c) P (P ({1}));
d) P ({9} ∪ P ({1})); e) {9, {2}} ∪ {{9}, 1}; f ) {9, {2}} ∩ {{9}, 1};
g) {1} ∈ {9, 1}; h) {1} ∈ {9, {1}}; i) {1} ∈ {(9, 1)};
10. Bestimmen Sie die folgenden kartesischen Produkte:
a){9, 5} × {5, 3} b){9, 2} × {2, 9}
c){{9}, {2}} × {{9}, {1}} d){9, {2}} × {{9}, 1}
e)({9, 3} × {3}) × {1, 2} f ){9, 3} × ({3} × {1, 2})
Sind die beiden Lösungen aus e) und f ) gleich ?
2
Eine Primzahl p ist eine natürliche Zahl (p ∈ N), die genau zwei unterschiedliche natürliche
Zahlen als Teiler hat: die eins und sich selbst.
34
ANHANG A. ÜBUNGSAUFGABEN
11. Gegeben sei Grundmenge G := N die Menge der positiven ganzen Zahlen
und es sei:
A := {2, 4} B := {1, 5} C := {a, c}
(a) Geben Sie alle Elemente der folgenden Mengen an:
(A×B)∪C
P({1, 2, 3, 4, 5}\B)
(A∩B)∪C
(A∪B)∩C
A∪(B∩C)
(b) Bestimmen Sie die Kardinalität der folgenden Mengen:
|P(A) ∪ P(B)|
|P(C) ∪ C|
|P(A) × P(B)|
|P(C) × C|
12. (? Punkte)
Die Mengen M1 , M2 und M3k , k ∈ N seien wie folgt definiert:
M1 := {1, 3, 5, 7},
M2 := {2, 3, 7, 11},
M3k := {−k, −(k − 1), . . . , −1, 0, 1, . . . , k − 1, k}
Bestimmen Sie jeweils die Menge M mit
1
a)M := 2|M3 |
b)M := M1 \ M2
e)M := M1 ∪ M2
c)M := M2 \ M1
f )M :=
∞
\
M3k
d)M := M1 ∩ M2
g)M :=
k=1
∞
[
M3k
k=1
13. Definieren Sie die folgenden Symbole formal in Zeichen:
(a) die Ungleichheit zweier Mengen 6=
(b) die Relation „A ist echte Teilmenge von B“: A ⊂ B, wenn A ⊆ B gilt
und es wenigstens ein x ∈ B gibt, das nicht zu A gehört.
14. Sei M := {2 · n| 3 ≥ n ∈ N} und N := {x ∈ Z| 4 > |x|}.
(a) Geben Sie alle Elemente der Mengen M, N, M ∩ N, M \ N, N \ M und
P (M ∩ N ) in aufzählender Form an.
(b) Geben Sie alle Elemente der Menge P (P (P (∅))) an.
(c) Sind die Mengen M ∪ N und M ∩ N disjunkt ?
(d) Sind die Mengen M ∩ N und (M ∩ N ) × N disjunkt ?
15. Mengen (? Punkte)
Es seien M := {1, 3, 5, −3, −1} und N := {p ∈ N|p Primzahl, p ≤ 10}.
Bestimmen Sie (P sei dabei die Potenzmenge)
|M |
|N |
(M \ N ) × (N \ M )
P (M \ N )
|P (M ∪ N )|
A.2. ÜBUNGEN GRUNDLAGEN
35
16. Die Menge A ∗ B der Elemente, die entweder zu A oder zu B gehören (ausschließendes oder), wird als Symmetrische Differenz bezeichnet. Nehmen Sie
eine formale Definition mit logischen Zeichen vor. Nehmen Sie dann eine
formale Definition mit ∪, ∩, \ vor. Ist A ∗ B = B ∗ A, also ist ∗ kommutativ
?
17. Es seien A, B Mengen mit A ⊆ B. Zeigen Sie: A ∪ B = B.
Es seien C, D Mengen mit C ∪ D = D. Zeigen Sie: C ⊆ D.
18. Wandeln Sie die folgenden beiden Terme T (mit Hilfe der Gesetze) so um,
dass
(a) T = A−1 ∪ (B −1 ∪ C) keine ∪ enthält.
(b) T = (A−1 ∪ B) ∩ (C −1 ∩ D) keine ∩ enthält.
19. Ein Element E heißt Neutralelement bzgl. einem Operator ◦, wenn für alle
anderen Elemente M gilt: M ◦E = M . Beispiel: die Null ist Neutralelement
bzgl. +, denn es gilt für alle Zahlen x: x+0 = x. Die Eins ist Neutralelement
bzgl. *, denn es gilt für alle Zahlen x: x ∗ 1 = x.
Sowohl ∩ als auch ∪ auf der Potenzmenge P (M ), d.h. das ist unsere Grundmenge, besitzen ein Neutralelement. Wie lauten diese ? (Begründung der
Antwort!)
20. Mengen (? Punkte)
Von 100 Personen besitzen 77 einen PC, einen Laptop oder ein Handy. 23
besitzen also keinen der drei Gegenstände. Einen eigenen PC besitzen 38
Personen, einen eigenen Laptop besitzen 14 Personen, ein Handy haben 67
Personen. 8 Personen besitzen PC, Laptop und Handy. Wie viele Personen
besitzen mindestens 2 der 3 Gegenstände ? Wie viele Personen besitzen
genau 2 der 3 Gegenstände ?
21. Mengen (? Punkte)
Von 27 Schülern einer Klasse gehen 18 gern in die Disco. Drei von diesen
18 lieben auch klassische Musik. Außer den 18 gibt es auch welche, die
nur klassische Musik mögen, und andere, die jede Art von Musik ablehnen.
Sieben Schüler lieben klassische Musik.
(a) Wie viele Schüler lieben nur klassische Musik ?
(b) Wie viele Schüler mögen weder klassische noch leichte Musik ?
22. Mengen (? Punkte)
Von 24 Studierenden eines Semesters spielen 15 Volleyball, 15 Handball und
10 Baketball. 5 Studierende spielen Volleyball und Basketball, 7 Handball
und Basketball, 3 spielen nur Handball und 4 betreiben alle drei Sportarten.
36
ANHANG A. ÜBUNGSAUFGABEN
(a) Wie viele Studierende spielen Volleyball und Handball ?
(b) Wie viele Studierende spielen nur Volleyball ?
(c) Wie viele Studierende spielen nur Basketball ?
(d) Wie viele Studierende betreiben keine der drei Sportarten ?
23. Mengen (? Punkte)
Das Spielfeld des Glücksspiels Roulette ist in drei Farben aufgeteilt: Rot,
Schwarz und Grün. Alle Felder haben zudem eine natürliche Zahl und können als Menge aufgeführt werden.
• Menge Zahlen auf rotem Grund
R = {1; 3; 5; 7; 9; 12; 14; 16; 18; 19; 21; 23; 25; 27; 30; 32; 34; 36}.
• Menge Zahlen auf schwarzem Grund
S = {2; 4; 6; 8; 10; 11; 13; 15; 17; 20; 22; 24; 26; 28; 29; 31; 33; 35}.
• Menge Zahlen auf grünem Grund
G = {0}.
Es gibt nun die Möglichkeit sein Geld auf Rot, Schwarz, Gerade, Ungerade,
einzelne Zahlen etc. zu setzen. Sie nehmen an diesem Spiel teil und setzen
jeweils den gleichen Betrag auf Rot und Gerade. Das bedeutet, bleibt die Kugel auf einer schwarzen ungeraden Zahl liegen verlieren Sie. Vereinfachend
gilt, gewinnen Sie, gibt es den Einsatz doppelt zurück und 0 zählt zu den
ungeraden schwarzen Zahlen. Geben Sie nun die Verknüpfung der Mengen
für folgende Ereignisse an und zählen die dazugehörigen Zahlen auf.
(a) Sie verlieren den gesamten Einsatz.
(b) Sie machen keinen Gewinn.
(c) Sie machen keinen Verlust.
(d) Sie machen weder Gewinn noch Verlust.
(e) Sie machen einen Gewinn.
24. Eine Fertigungsserie von Schokoladentafeln wurde auf Abmessung und Verarbeitung geprüft. Von den insgesamt 140 fehlerhaften Tafeln hatten 90 Tafeln Abmessungsfehler, 20 waren sowohl in den Abmessungen als auch in
der Verarbeitung fehlerhaft.
(a) Lösen Sie die Aufgabe (b), indem Sie zunächst den allgemeinen Zusammenhang zwischen |A|, |B|, |A ∩ B| und |A ∪ B| finden, d.h. eine
Gleichung aufstellen, in der alle vier Terme vorkommen. Geben Sie
eine kurze Begründung ab, warum die Gleichung richtig ist.
A.2. ÜBUNGEN GRUNDLAGEN
37
(b) Wie viele Schokoladentafeln hatten Verarbeitungsfehler, und wie groß
war die Anzahl der Schokoladentafeln, die ausschließlich fehlerhaft in
der Abmessung verarbeitet worden waren ? (Verwenden Sie die Gleichung aus (a) ; andere Berechnungen werden nicht akzeptiert bzw.
bewertet!)
(c) Erweitern Sie die gefundene Beziehung auf drei Mengen A, B, C.
25. Zeigen Sie die Gültigkeit des allgemeinen de-Morgan’sche Gesetz
(
n
[
i=1
Ai )
−1
=
n
\
(Ai )−1
i=1
26. Wie lautet die kürzeste (d.h. mit einem Minimum an Zeichen schreibbare)
Form und die kanonische disjunktive Normalform des nachfolgenden
Terms ?
T = A ∪ (A−1 ∩ B) ∪ (A−1 ∩ B −1 ∩ C)
Für die kürzeste Form sind mindestens 5 Zeichen notwendig: 3 Mengensymbole und 2 binäre Mengenoperatorensymbole. Die kannonische disjunktive
Normalform benötigt 7 Disjunktionsglieder. Der Lösungsweg ist anzugeben,
z.B. durch Angabe der Gesetze der Mengenoperationen, die bei der Transformation des Terms T verwendet wurden. Die Transformation ist notwendig,
um zu zeigen, dass die Terme gleich sind!
Eine kanonische disjunktive Normalform ist eine disjunktive Normalform
(z.B. T ist in disjunktiver Normalform), in der in jedem Disjunktionsglied
alle verwendeten Mengen (oder deren Inversen) vorkommen (hier also A,
B und C oder deren Inversen), wie z.B. in T bereits im Disjunktionsglied
(A−1 ∩ B −1 ∩ C) vorhanden. Eine Disjunktion wird in der Mengenlehre mit
∪ und eine Konjunktion mit ∩ assoziiert.
27. Die Grundmenge sei N. Gegeben seien folgende „Objekte“:
A = (1, 2); B = 1; C = {1, 2}; D = {{1, 2}}; E = {(1, 2)};
F = ({1, 2}); G = ((1, 2))
Nennen Sie alle ∈ und ⊆ Beziehungen
a) der „Objekte“ untereinander.
b) der „Objekte“ zu der Potenzmenge P (N).
28. a) Beschreiben Sie die Menge M durch Aufzählung, die durch
M := {x ∈ N|x ≤ 2} definiert ist (N := Menge der natürlichen Zahlen).
b) Geben Sie die Potenzmenge P (M ) an.
c) Stimmt nachfolgende Behauptung ? Begründen Sie Ihre Entscheidung!
(M ∩ P (M )) ∈ P (M )
29. Mengen (? Punkte)
Es seien A, B, C, D, E Mengen und A = {x ∈ R|2 ∗ x2 = 8}. Zeigen Sie:
38
ANHANG A. ÜBUNGSAUFGABEN
(a) {2} =: B ⊂ A,
(b) {−2, 2} =: C = A,
(c) Q =: D ⊃ A
(d) Es sei E eine Menge mit |E| = n, e 6∈ E. Um wie viele Elemente ist
die Potenzmenge P (E ∪ {e}) größer als die Potenzmenge P (E)?
wobei X := Y bedeutet, dass X (die Seite mit Doppelpunkt) definiert wird
durch Y .
30. Quantoren (? Punkte)
Sei M := {2 ∗ n|n ∈ N}. Zeigen Sie, dass die Aussage
√
∃x ∈ M mit 2 x ∈ (N \ M )
falsch ist.
31. Quantoren (? Punkte)
Zeigen Sie durch ein Gegenbeispiel, dass die beiden Aussagen
(∀x ∈ M : A1 (x)) ∨ (∀x ∈ M : A2 (x)) ↔ (∀x ∈ M : (A1 (x) ∨ A2 (x)))
(∃x ∈ M : A1 (x)) ∧ (∃x ∈ M : A2 (x)) ↔ (∃x ∈ M : (A1 (x) ∧ A2 (x)))
im Allgemeinen nicht gelten, d.h. definieren Sie eine Menge M und Aussagen A1 (x), A2 (x), so dass die beiden obigen Aussagen falsch sind. Beispiel:
M := {1, 2, 3}; A1 (x) := x ≤ 4; A2 (x) := x ≥ 0
Aufgabe A.4 (Relationen)
1. Man zeige durch Angabe eines Beispiels, dass
die „Mitgliedschaftsrelation“ ∈ nicht transitiv ist.
2. Es sei A = B = {1, 2, 3, 4} =: M . Zwei Elemente x, y ∈ M mögen genau
dann in Beziehung zueinander stehen, wenn ihr größter gemeinsamer Teiler
gleich 1 ist. Wie lautet die Menge R ⊆ A × B in aufzählender Form ?
3. Es sei {1, 2, 3, 4, 5, 6, 7, 8, 9, 0} =: M . Zwei Elemente x, y ∈ M mögen genau
dann in Beziehung zueinander stehen, wenn ihre Summe gleich 9 ist. Wie
lautet die Menge R ⊆ M × M in aufzählender Form ?
4. Es sei {0, 1} =: M . Geben Sie M 3 := M × M × M an!
5. Sei P = {−1, 0, 1}, R = {(a, b, c)|a, b, c ∈ P ∧ a2 + b2 = c2 }. Geben Sie R
in aufzählender Form an.
6. Welche Formel gilt für die Anzahl der Elemente eines endlichen kartesischen
Produkts M1 × . . . × Mn ?
A.2. ÜBUNGEN GRUNDLAGEN
39
7. Ein Theater verfüge über 27 Reihen zu je 19 Plätzen. Jeder Platz ist durch
seine Reihennummer r ∈ R und seine Sitznummer s ∈ S, also das Paar
(r, s) ∈ R×S eindeutig festgelegt (Eintrittskarte!). Im Rahmen einer Systembeschreibung werde jede Vorstellung V als Menge der verkauften Plätze verstanden. Es ist also V ⊆ R×S. Beschreiben Sie damit folgende Sachverhalte
formal:
(a) die Vorstellung ist ausverkauft;
(b) keine Eintrittskarte wurde verkauft;
(c) die Menge aller möglichen Vorstellungen (bezogen auf die Eintrittskarten!)
(d) von jeder Reihe wurde mindestens ein Platz verkauft;
(e) wenigstens eine Reihe ist vollständig besetzt.
8. Die Relation R ⊆ G2 ; G = {1, 2, 3} und R = {(1, 1), (1, 2), (2, 1), (2, 2)} ist
symmetrisch und transitiv. Warum ist sie dennoch nicht reflexiv ?
9. Gegeben seien die Äquivalenzklassen {{a, b}, {c}, {d}} auf der Menge M =
{a, b, c, d}. Geben Sie die dadurch bestimmte Äquvivalenzrelation R auf M
durch Aufzählung der Elemente an!
10. Relationen (? Punkte)
Gegeben sei die Menge M der Studierenden M := { Christoph, Dietrich,
Hans, Karl, Pius, Sophie}. Hans kennt Sophie schon von Kindesbeinen
an. Karl und Dietrich hatten schon in ihrem Erststudium miteinander zu
tun. Christoph kennt Hans aus einem früheren Semester. Pius vertraut nur
sich selbst.
(a) Bilden Sie die oben geschilderten Sympathie-Verhältnisse als Relation
R1 ab. Identifizieren Sie dabei nicht mehr Beziehungen, als wie durch
obigen Text ausgedrückt werden!
(b) Erweitern Sie R1 zu einer Äquivalenzrelation R2 , indem Sie die entsprechenden Abschlüsse bilden, d.h. die für eine Äquivalenzrelation
noch minimal benötigten Elemente hinzufügen. Dokumentieren Sie,
welche Eigenschaft einer Äquivalenzrelation welche hinzugefügten Elemente bedingt hat.
(c) Berechnen Sie die Quotientenmenge, d.h. die Menge aller Äquivalenzklassen, bezüglich R2 .
11. Relationen (? Punkte)
Gegeben sei die Menge M = {a, b, c, d} und die Relation
M × M ⊇ R = {(a, a), (a, b), (b, c), (d, b), (c, d)}.
40
ANHANG A. ÜBUNGSAUFGABEN
(a) Ist R reflexiv? Wenn nein (Begründung!), erweitern Sie R (wenn möglich) zu einer reflexiven Relation R1 mit R ⊆ R1 , R1 6= M × M und
R1 reflexiv.
(b) Ist R symmetrisch? Wenn nein (Begründung!), erweitern Sie R (wenn
möglich) zu einer symmetrischen Relation R2 mit R ⊆ R2 , R2 6=
M × M und R2 symmetrisch und es handelt sich um eine minimal
notwendige Erweiterung.
(c) Ist R transitiv? Wenn nein (Begründung!), erweitern Sie R (wenn
möglich) zu einer transitiven Relation R3 mit R ⊆ R3 , R3 6= M ×
M und R3 transitiv und es handelt sich um eine minimal notwendige
Erweiterung.
12. Sei M = {1, 2, 3, 4, 5, 6}, R = {(x, y)|x ist echter Teiler (6= 1, 6= y) von y}.
Wie muß die Relation R zu der Halbordnungsrelation R′ erweitert werden
? Geben Sie R′ in aufzählender Form an.
13. Relationen (? Punkte)
(a) Ist folgende Relation eine (strenge/nicht strenge) Totalordnung oder
eine partielle Ordnung ?
R := {(X, Y ) ∈ P (M ) × P (M )|X ist eine Teilmenge von Y } auf der
Potenzmenge P (M ) aller Teilmengen der Menge M .
(b) Ist folgende Relation eine (partielle/totale) Halbordnung oder eine (partielle/totale) strenge Ordnung ?
R := {(X, Y ) ∈ N × N|X ist Teiler von Y }.
(c) Gesucht ist eine Relation auf einer Menge M , welche reflexiv, transitiv, symmetrisch und antisymmetrisch ist. Kann eine solche Suche
überhaupt erfolgreich sein ?
(d) Wie viele binäre Relationen sind auf einer m-elementigen Grundmenge
(m ∈ N) definierbar ?
14. Gegeben sei folgende Definition:
Eine Wohlordnung einer Menge M ist eine totale Ordnung,
bei der jede nichtleere Teilmenge von M ein kleinstes Element
bezüglich dieser Ordnung hat.
(a) Auf N × N definiert man eine totale Ordnung (das brauchen Sie nicht
zu zeigen!) durch (m, n) (p, q) :↔ m ≤ p und (m = p) → (n ≤ q);
wobei ≤ die natürliche Ordnung auf N bezeichnet. Zeigen Sie, dass eine Wohlordnung ist. Sie können davon ausgehen, dass alle Teilmengen von N wohlgeordnet sind.
A.2. ÜBUNGEN GRUNDLAGEN
41
(b) Sei eine totale Ordnung auf einer Menge M , wie unter (a) gegeben,
und ≺ die zugehörige strikte Ordnung. Zeigen Sie, dass die Bedingung
∀x ∈ M : {y ∈ M |y ≺ x} ist endlich
hinreichend, aber nicht notwendig dafür ist, dass ≺ eine Wohlordnung
ist.
15. Hasse-Diagramm (? Punkte)
Gegeben sei folgende Definition:
Das Hasse-Diagramm für eine Halbordnung (Partialordnung)
(M, ≤) ist ein gerichteter Graph, wobei die Elemente von M die
Knoten bilden. Zwei Knoten a und b werden durch eine Kante (a ← b) verbunden, wenn a < b gilt und es kein c gibt mit
a < c < b. Die Richtung der Kante wird dadurch zum Ausdruck
gebracht, dass sich der Knoten b oberhalb von a befindet. Das
Hasse-Diagramm ist zyklenfrei: Schleifen bei Reflexivität werden
weggelassen.
Es sei nun (M ; ≤) eine partialgeordnete Menge mit M = {1; 2; . . . ; 8} und
mit der Relation ≤ definiert als x ≤ y genau dann, wenn x|y (x teilt y). Bilden Sie das Hasse-Diagramm von (M ; ≤). Bilden Sie dann alle maximalen
Vergleichsketten bzgl. der Halbordnung ≤.
16. Halbordnung (? Punkte)
Es sei R ⊆ A × A eine Halbordnung (bzw. partielle Ordnung) und K ⊆ A
eine Teilmenge von A. Wir definieren:
Ein Element a ∈ K heißt genau dann minimales Element von
K (bezüglich R), wenn ∀b ∈ K gilt: Ist b ≤R a (bzw. (b; a) ∈ R),
so ist a = b. (Mit anderen Worten: Das Element a ∈ K ist ein
minimales Element von K genau dann, wenn es kein b ∈ K mit
b ≤ a gibt, das bezüglich R kleiner als a ist.)
Bestimmen Sie jeweils für folgende Halbordnungen R ⊆ A × A über der
Grundmenge A alle minimalen Elemente von K ⊆ A:
(a) A := N, R := {(m; n)|m ≤ n} und K := {n|n > 1 ∧ n ist Primzahl }
(b) A := Z, R := {(m; n)|m ≤ n} und K := Z
(c) A := N, R := {(m; n)|m teilt n} und K := N \ {1}
17. Halbordnung (? Punkte)
Es sei M = {{1, 2, 3}, {0, 2}, {1}, {0, 1, 2}, {2, 3}, {0, 1, 2, 3}, {0, 2, 3}}. Die
Menge M ×N0 wird durch folgende Ordnungsrelation zu einer Halbordnung:
(S1 , k1 ) (S2 , k2 ) genau dann wenn S1 ⊆ S2 und k1 ≤ k2
42
ANHANG A. ÜBUNGSAUFGABEN
Betrachten Sie die folgenden vier Teilmengen von M × N0 :
M1 = {(S, k)k1 ∈ S, k ∈ S}
M3 = {(S, k)k2 ∈ S, k 6∈ S}
M2 = {(S, k)k1 ∈ S, k 6∈ S}
M4 = {(S, k)k1 ∈ S, k = |S|}
Geben Sie zu jeder dieser Mengen das (sofern es existiert) bzgl. der gegebenen Halbordnung kleinste Element (Infimum ) und größte Element (Supremum) an.
18. Es sei die Relation R wie folgt definiert:
R := {(x, y)|∃λ ∈ Z : x = y + 3 ∗ λ} wobei Z := {. . . − 2, −1, 0, 1, 2, . . .}
Ist R eine Äquivalenzrelation ?
19. Äquivalenzrelation (? Punkte)
Es seien die nachfolgenden Relationen Ri gegeben. Welche der Relationen
sind Äquivalenzrelationen ?
(a)
R1 := {(x, y) ⊆ Z × Z | x = y oder x = y + 1 oder x = y − 1}
(b)
R2 := {(x, y) ⊆ (Z \ {0}) × (Z \ {0}) | x ∗ y > 0}
(c)
R3 := {(x, y) ⊆ Z × Z | 2 teilt x und 4 teilt y}
20. Äquivalenzrelation (? Punkte)
Es sei M = {1, 2, . . . , 10}. Sind nachfolgende Relationen Äquivalenzrelationen ? Geben Sie für die Äquivalenzrelationen die Äquivalenzklassen an!
(a)
R1 = {(m, n) ∈ M × M k∃k ∈ N mit
m
= 2k}
n
(b)
R2 = {(m, n) ∈ M × M k∃k ∈ N mit m + n = 2k}
Tipp: Wenn m + n = 2k, dann ist für m 6= n m > k, n < k oder m < k, n >
k.
A.2. ÜBUNGEN GRUNDLAGEN
43
21. Äquivalenzrelation (? Punkte)
(a) Sei A = {1, 2, 3}. Geben Sie alle möglichen Äquivalenzrelationen über
A × A in aufzählender Form an.
(b) Sei A die Menge aller Geraden im Raum (R3 ) und
R = {(x, y) ∈ A × A|x, y windschief oder x = y}. Ist R eine Äquivalenzrelation ?
Tipp: Man nennt zwei Geraden windschief, wenn sie sich weder schneiden noch parallel zueinander sind.
(c) Sei R = {(x, y) ∈ Z × Z|∃n, m ∈ N : x − y = 5n + 7m}. Zeigen Sie: R
ist transitiv.
22. Äquivalenzrelation (? Punkte)
Zeigen Sie, dass nachfolgende Relationen Äquivalenzrelationen sind. Beschreiben Sie zudem ihre Äquivalenzklassen und geben Sie die dadurch verursachte Zerlegungen der Ursprungsmenge (z.B. N0 ) an.
(a)
≡4 := {(a, b)|a, b ∈ N0 , ∃k ∈ Z : a − b = 4 ∗ k}
(b)
∼LZ := {(a, b)|a, b ∈ N, a und b haben die selbe letzte Ziffer}
23. Äquivalenzrelation (? Punkte)
R ⊆ M × M und S ⊆ M × M seien zwei beliebige Äquivalenzrelationen
auf einer Menge M . Welche der folgenden Relationen sind dann ebenfalls
Äquivalenzrelationen ? Begründen Sie Ihre Antwort durch Nachweis der
benötigten Eigenschaften (reflexiv, symmetrisch und transitiv) oder durch
Angabe eines Gegenbeispiels.
(a) R ∪ S
(b) R ∩ S
24. Relationen (? Punkte)
Begründen Sie: Ist R ⊆ M × M eine reflexive Relation und S ⊆ M × M
eine beliebige Relation, dann ist R ∪ S reflexiv auf M.
Würde dies jeweils auch für die Eigenschaften symmetrisch und transitiv
von R gelten ? Begründen Sie Ihre Antwort durch Nachweis der jeweiligen
Eigenschaften oder durch Angabe eines Gegenbeispiels.
44
ANHANG A. ÜBUNGSAUFGABEN
25. Äquivalenzrelation (? Punkte)
Es sei M = {x ∈ N|x < 11} und R eine Relation mit
M × M ⊇ R = {(a, b)|2 ist gleich oft Primfaktor von a und b}
, d.h. beide sind durch dieselben Zweierpotenzen teilbar.
(a) Zeigen Sie, das R eine Äquivalenzrelation auf M ist.
(b) Bestimmen Sie den Quotienten, d.h. die Menge aller Äquivalenzklassen
von R.
26. Kuratowski (ein berühmter Theoretiker) hat gezeigt, dass eine zweistellige
Relation auf den Mengenbegriff zurückgeführt werden kann. Man setze dazu (a, b) := {{a}, {a, b}}, d.h. das Tupel (a, b) wird dargestellt als Menge
{{a}, {a, b}}. Beachten Sie, dass das Element {a} die Reihenfolge kodiert,
da Elemente in einer Menge in keiner Reihenfolge stehen.
Zeigen Sie, dass folgende Eigenschaften dann noch gelten:
(a) (a, b) 6= (b, a) für a 6= b
(b) (a, b) = (c, d) ↔ a = c ∧ b = d
27. Vorgelegt sei die Relation R ⊂ N2 mit (x, y ∈ N)
R = {(2, 3), (3, 4), (x, 4), (1, y)}
(a) Welche Eigenschaft hat R bei jeder Belegung von x, y bzw. welche hat
R nicht ?
(b) Wie sind x und y zu belegen, damit R asymmetrisch wird?
(c) Mit welcher Belegung von x, y erreichen Sie, dass R transitiv wird?
28. Auf der Menge M = R × R erklären wir: Zwei Zahlenpaare (a, b) und (c, d)
aus M sollen genau dann einer Relation R angehören, wenn zwischen ihnen
die Gleichung a2 + b2 = c2 + d2 gilt.
(a) Zeigen Sie, dass R ⊆ M 2 eine Äquivalenzrelation ist!
(b) Wie lauten die Äquivalenzklassen ?
(c) Was ist die geometrische Bedeutung der Menge aller Äquivalenzklassen ?
29. Wie lautet die Menge aller Äquivalenzklassen der (Äquivalenz-)Relationen
R1 bzw. R2 auf einer Menge M , welche die feinste (R1 ) und die gröbste
(R2 ) Zerlegung von M in Klassen bewirkt ? Die feinste Zerlegung hat die
maximal mögliche Anzahl an Äquivalenzklassen und die gröbste Zerlegung
die minimalste Anzahl an Äquivalenzklassen.
A.2. ÜBUNGEN GRUNDLAGEN
45
30. Gegeben sei die folgende Aufteilung P = P1 , P2 einer Menge M = P1 ∪ P 2,
mit P1 = {Wombat, Blauwal} und P2 = {Huhn, Barsch, Zitteraal}. Geben
Sie die Elemente der Äquivalenzrelation R ⊆ M × M zu der gegebenen
Aufteilung P an, d.h. geben Sie R in aufzählender Form an, wobei die Pi
die jeweilige Äquivalenz angeben!
31. Sei M = {Hanna, Otto, Susanne, Fridtjof, Anna} Eine Person aus M
steht zu einer Person aus M in der Relation R, wenn sie dasselbe Geschlecht
hat oder (im Sinne von „∨“) ihre Namen gleich viele Buchstaben. (Hinweis:
Die kursiven sind männlich, die anderen weiblich)
(a) Geben Sie alle Elemente der Relation R explizit durch Aufzählung an!
(b) Ist R eine Äquivalenzrelation oder eine Ordnungsrelation oder eine
Abbildung ? Geben Sie für alle drei Eigenschaften eine Begründung
an!
(c) Eine Eigenschaft, die in (b) nicht erfüllt ist, kann man sehr leicht
erfüllen, indem man eine Person aus M streicht. Welche Eigenschaft
ist das und welche Person muss man streichen ?
Aufgabe A.5 (Abbildungen)
1. Welche der folgenden Relationen sind Abbildungen ? (Begründen Sie kurz Ihre Antwort.)
(a) R1 = {(x, y)|x ∈ R ∧ x 7→ y = sin(x)}
(b) R2 = {(x, y)|x ∈ R ∧ x 7→ y = tan(x)}
(c) R3 = {(x, y)|x ∈ [−1; 1] ∧ x 7→ y =
1
}
x2 −1
(d) R4 = {(x, y)|x ∈ R+ ∧ x 7→ y = ln(x)}
(e) R5 = {(x, y)|x ∈ [−5; 5] ∧ x2 + y 2 = 25}
(f ) R6 = {(x, y)|x ∈ R ∧ x 7→ y = −x}
2. Wie hat man eine Relation, die keine Abbildung ist, zu verändern, damit sie eine Abbildung wird ? Wenden Sie dieses Verfahren auf die nichtfunktionellen Relationen der vorangegangenen Aufgabe an.
3. Gegeben sind die Mengen A = {1, 2, 3, 4} und B = {a, b, c}. Die Abbildungen f : A → B und g : B → A sind definiert durch
f := {(1, b), (2, b), (3, c), (4, a)} und g := {(a, 1), (b, 2), (c, 3)}
Geben Sie die Abbildung der Komposition g ◦ f : A → C in aufzählender
Form an.
4. Abbildungen (? Punkte)
Seien A := {m, a, f, i} und B := {a, r, k}.
46
ANHANG A. ÜBUNGSAUFGABEN
(a) Welche der Relationen Ri (i ∈ {1, 2, 3, 4}) mit Typ A × B sind eine
Abbildung bzw. keine Abbildung ? (Begründung/Lösungsweg für jede
Relation nicht vergessen!)
R1 := {(m, a), (a, a), (f, r), (i, k)}
R2 := {(m, a), (f, a), (a, a), (i, a)}
R3 := {(m, a), (a, r), (i, k), (a, a)}
R4 := {(m, r), (a, k), (f, a), (f, k)}
(b) Geben Sie für die Relationen Ri , die eine Abbildung sind, an, welche
der Eigenschaften injektiv, surjektiv und/oder bijektiv sie besitzen bzw.
sie nicht besitzen. (Begründung/Lösungsweg nicht vergessen!)
5. Abbildungen (? Punkte)
Es sei f : R → R mit f√
(x) = x2 , also x 7→ x2 . Die ggf. mögliche Umkehrfunktion ist f −1 (x) = ± 2 x.
(a) Zeigen Sie, dass f weder injektiv noch surjektiv ist.
(b) Finden Sie Teilmengen U, V ⊆ R mit f (U ) ⊆ V so, dass f ′ : U → V
mit f ′ (x) = x2 injektiv, aber nicht surjektiv ist.
(c) Finden Sie Teilmengen U, V ⊆ R mit f (U ) ⊆ V so, dass f ′ : U → V
mit f ′ (x) = x2 surjektiv, aber nicht injektiv ist.
(d) Finden Sie Teilmengen U, V ⊆ R mit f (U ) ⊆ V so, dass f ′ : U → V
mit f ′ (x) = x2 bijektiv ist.
6. Abbildungseigenschaften (? Punkte)
Bestimmen Sie jeweils L und R, mit L, R ∈ {{a, b, c}, {a, b}, {a}}, so dass
mit f (x) = x eine Abbildung f : L → R definiert ist, die
(a) injektiv, aber nicht surjektiv ist.
(b) bijektiv ist.
(c) konstant und surjektiv ist.
(d) konstant und nicht surjektiv ist.
7. (? Punkte)
Es seien A = {1; 2; 3; 4; 5} und B = {rot; gelb; gruen; blau; schwarz}. Betrachten Sie die folgende Relation R ⊆ A × B:
R := {(1; rot); (1; gelb); (1; blau); (2; gelb); (2; blau); (3; rot); (3; gruen); (3; schwarz);
(4; rot); (4; gelb); (4; gruen); (4; schwarz); (5; rot); (5; gruen); (5; schwarz)}
(a) Zeigen oder widerlegen Sie, dass es eine bijektive Abbildung f : A → B
mit f ⊆ R gibt.
A.2. ÜBUNGEN GRUNDLAGEN
47
(b) Zeigen oder widerlegen Sie, dass es eine bijektive Abbildung f : A → B
mit f ⊆ (A × B) \ R gibt.
8. Sei A := {x ∈ R|x 6= 1} und f : A → A definiert durch:
x
f (x) :=
x−1
Zeigen Sie, dass f bijektiv ist und bestimmen Sie die inverse Abbildung f −1
von f .
9. Welche der Eigenschaften injektiv und surjektiv, ggf. bijektiv, treffen auf
die nachfolgende Abbildung h : AM atrix 7→ N0 , mit ni ∈ N0 zu ? Sollte
die Abbildung h surjektiv oder bijektiv sein, geben Sie bitte eine inverse
Abbildung an und geben Sie an, ob diese eindeutig ist. Begründen Sie jeweils
Ihre Ergebnisse!
4
X
n1 n2
h(
)=
ni
n3 n4
i=1
10. Bezeichne B die Menge aller Bundesbürger, K die Menge aller amtlichen
KFZ-Kennzeichen. B × K ⊇ R = {(x, y)|x hält KFZ mit Kennzeichen y}.
R ist keine Abbildung! (warum ?) Konstruieren Sie Teilmengen B ∗ ⊆ B
und K ∗ ⊆ K, und damit R∗ ⊆ R, so dass f : B ∗ → K ∗ mit x 7→ (y =
f (x)) ⇔ R∗ xy) eine bijektive Abbildung darstellt!
11. Es seien die Abbildungen f1 und f2 gegeben, wobei f1 : { Menge aller Ehefrauen } → { Menge aller Ehemänner } und f2 : { Menge aller Menschen
} → { Menge aller Mütter }. Es wird hier angenommen, dass die Mütter
alle „Ehemänner“ haben und daher auch alle „Ehefrauen“ sind!
• x 7→ f1 (x) bedeute „f1 (x) ist Ehemann von x“;
• x 7→ f2 (x) bedeute „f2 (x) ist Mutter von x“;
Bilden Sie die Verkettungen f1 ◦ f2 ; f2 ◦ f1 ; f2 ◦ f2 an und geben Sie jeweils
eine verbale Beschreibung dieser Verkettung an, etwa in der Form „... ist
Onkel von x“.
12. Die Abbildung rn : N0 → B sei definiert durch rn (x) := Rest der Division
von x durch n und 1 < n ∈ N. Z.B. r6 (12) = 0; r6 (13) = 1; r6 (257) = 5
(a) Wie sieht jeweils (für die unterschiedlichen n) der Bildbereich B aus ?
(b) Prüfen Sie für rn die Eigenschaften injektiv, surjektiv und bijektiv
nach.
(c) Betrachten Sie die Relation Rn := {(x, y)|x, y ∈ N0 ∧ rn (x) = y}.
Prüfen Sie die Eigenschaften reflexiv, symmetrisch, transitiv, asymmetrisch, und antisymmetrisch nach. Ist Rn eine Äquivalenzrelation ?
48
ANHANG A. ÜBUNGSAUFGABEN
(d) Sei f eine Abbildung von A in B, also f : A → B, mit
A = {(x1 , x2 )|(x1 , x2 ) ∈ R × R}
B = {(y1 , y2 )|(y1 , y2 ) ∈ R × R}
und (x1 , x2 ) 7→ (y1 , y2 ) gemäß
y1 = a11 x1 + a12 x2
∀aik ∈ R :
y2 = a21 x1 + a22 x2
Unter welchen Bedingungen für die aik ist f eine Bijektion ? Wie lautet
in diesem Fall die umgekehrte Elementzuordnung (y1 , y2 ) 7→ (x1 , x2 ) ?
Aufgabe A.6 (Strukturen)
1. Untersuchen Sie
(a) die Arithmetische-Mittel-Verknüpfung m auf R
m : R × R → R mit (a, b) 7→ 21 (a + b)
(b) die Maximum-Verknüpfung max auf N0
maxN0 × N0 → N0 mit (a, b) 7→ a falls a ≥ b, sonst b
jeweils auf Kommutativität, Assoziativität, Neutralelement und Idempotenz.
2. Sowohl ∩ als auch ∪ auf der Potenzmenge P (M ) besitzen ein Neutralelement. Wie lauten diese ?
3. Es sei ⋆ eine zweistellige nicht assoziative Operation. Dann ist, falls keine
zusätzlichen Vereinbarungen getroffen wurden, der Ausdruck
a⋆b⋆c⋆d
syntaktisch nicht korrekt, da z.B. sein „Wert“ bei einer Belegung der Variablen von der Art der Klammerung abhängt.
Notieren Sie alle möglichen Klammerungen für diesen Ausdruck, so dass
dieser zu einem syntaktisch korrekten Ausdruck wird. Berechnen Sie deren
Werte für a = 2, b = 1, c = 3, d = 2 mit ⋆ = Potenzierung, d.h. a ⋆ b := ab .
Aufgabe A.7 (Beweistechniken)
1. Untersuchen Sie ob gilt (für n ∈ N):
(a) (? Punkte)
n ∗ (n + 1)
∈N
2
(b) (? Punkte)
n ∗ (n + 1) ∗ (n + 2)
∈N
2∗3
A.2. ÜBUNGEN GRUNDLAGEN
49
2. (? Punkte)
Beweisen Sie mittels vollständiger Induktion:
23∗n + 13
∈ N0
7
3. (? Punkte)
Beweisen Sie mittels vollständiger Induktion:
72n − 2n
∈N
47
4. (? Punkte)
n
X
i3 = (
i=1
n ∗ (n + 1) 2
)
2
5. (? Punkte)
Zeigen Sie mittels vollständiger Induktion:
13 + . . . + n3 = (1 + . . . + n)2
Tipp: beachten Sie unbedingt die erste binomische Formel (
(a + b)2 = a2 + 2 ∗ a ∗ b + b2
P
) sowie die gaußsche Summenformel ( ni=1 i =
der 2 -Seite.
n∗(n+1)
)
2
und beginnen mit
6. (? Punkte)
∀n ∈ N∃k ∈ N : n ∗ (n2 + 5) = 3 ∗ k ,d.h. n ∗ (n2 + 5) ist durch 3 teilbar.
7. (? Punkte)
Es sei nachstehende rekursiv definierte Folge gegeben.
a1 =
1
1
an+1 = an +
2
(n + 1)(n + 2)
Behauptung:
an =
n
X
i=1
n
1
=
i(i + 1)
n+1
50
ANHANG A. ÜBUNGSAUFGABEN
8. vollständige Induktion (? Punkte)
Eine Folge ganzer Zahlen x1 , x2 , . . . , xn sei für n ∈ N definiert durch:
x1 = 1
und
xn+1 :=
xn
xn + 2
Beweisen Sie mittels vollständiger Induktion, dass xn =
Tipp: Es gilt
a
b
=
1
.
( ab )
1
2n −1
gilt.
9. Beweistechniken (? Punkte)
Die Folge an sei definiert durch
a0 = 2, a1 = 5 und an+1 = 5an − 6an−1
für n ∈ N0 .
(a) Zeigen Sie mittels vollständiger Induktion:
Für alle n ∈ N0 gilt an = 2n + 3n .
Tipp: Arbeiten Sie ausnahmsweise direkt ohne explizite Ermittlung des
Schrittes, d.h. setzen Sie die Induktionsbehauptung unmittelbar ein!
(b) Die Folge bn sei definiert durch
b0 = 2, und bn+1 = 2bn + 3n
Zeigen Sie mittels vollständiger Induktion:
Für alle n ∈ N0 gilt an = bn .
Tipp: Verwenden Sie dazu das Ergebnis aus Teil (a).
10. (? Punkte)
Berechnen Sie für n ∈ N:
32 − 34 + 36 − 38 + . . . + 34n−2 − 34n =?
Weisen Sie die Korrektheit mittels vollständiger Induktion nach.
11. (? Punkte)
Leiten Sie eine rekursive Formel an für die Funktion f (n) = 3 − n1 her und
überprüfen Sie Ihr Ergebnis durch vollständige Induktion.
12. Beweistechniken (? Punkte)
her
Leiten Sie eine rekursive Formel a(n) für die Funktion f (n) = n!−1
n!
und überprüfen Sie Ihr Ergebnis (gilt a(n) = f (n)?) durch vollständige
Induktion.
A.2. ÜBUNGEN GRUNDLAGEN
51
13. (? Punkte)
Bestimmen Sie die Methode zur direkten Berechnung der nachfolgenden Rekursionsgleichung. Weisen Sie die Korrektheit mittels vollständiger Induktion nach.
∀n ∈ N : a1 = 1 und an+1 = 2an + 1
Beispiel: zur Rekursionsgleichung a1 = 1; an+1 =
Berechnung.
an
an +2
ist
1
2n −1
die direkte
14. (? Punkte)
Zeigen Sie die so genannte Bernoullische Ungleichung:
(1 + x)n ≥ 1 + n ∗ x
15. (? Punkte)
Zeigen Sie die Allgemeingültigeit folgender Ungleichung:
∀n ≥ 2 :
√
n≤
n+1
2
16. (? Punkte)
n
X
i3 =
i=0
n2 ∗ (n + 1)2
4
17. (? Punkte)
Zeigen Sie mittels vollständiger Induktion, dass 2n > n2 gilt. Gehen Sie
zunächst davon aus, dass dies für alle n ∈ N gelten soll.
Im Induktionsschritt werden Sie jedoch bemerken, dass dies nicht sein kann.
Ab welchem n ∈ N gilt diese Ungleichung tatsächlich ?
18. (? Punkte)
Führen Sie unter Verwendung der Induktionsbehauptung nur den Induktionsschritt durch!
n
X
1
3n + 1
=
(2i − 1)(2i + 1)
2n + 1
i=0
Gibt es einen Induktionsanfang ?
19. (? Punkte)
Überprüfen Sie die Gültigkeit der nachfolgenden Aussage für n = 1, 2, 3, 4
und finden Sie ggf. den Fehler in nachfolgendem Induktionsbeweis.
Behauptung: Für alle n ∈ N gilt: 2∗n−1 ≤ 2n−1 . Beweis durch vollständige
Induktion:
52
ANHANG A. ÜBUNGSAUFGABEN
Induktionsanfang Für n = 1 ist 2 ∗ 1 − 1 = 1 = 20 = 21−1 .
Induktionsbehauptung Die Behauptung sei für ein beliebiges n gezeigt.
Induktionsschritt Wir bestimmen zunächst den Schritt: 2(n + 1) − 1 =
(2n − 1) + 2, d.h. der Schritt ist +2. Es gilt nun:
(2n − 1) + 2 ≤ 2n−1 + 2 ≤ 2n−1 + 2n−1 = 2n
und damit gilt die Behauptung.
20. (? Punkte)
∀n ∈ N, n > 7 : 3n > n4
√
Tipp: Zeigen Sie 3n+1 > 3 ∗ n4 > (n + 1)4 . 4 x können Sie bei der Ungleichung bedenkenlos verwenden.
21. (? Punkte)
Unter den drei Zahlen n − 2, n − 1, und n mit n ∈ N0 ∧ n ≥ 2, ist immer
eine Zahl, die durch 3 teilbar ist. (N0 = {0, 1, 2, . . .})
22. (? Punkte)
Führen Sie unter Verwendung der Induktionsbehauptung nur den Induktionsschritt durch!
Eine beliebige natürliche Zahl ist gleich ihrem direkten Vorgänger, d.h. ∀n ∈
N : n = (n − 1).
Gibt es einen Induktionsanfang ?
23. (? Punkte)
n
X
i=0
2i = 1 + 2 + 4 + . . . + 2n = 2n+1 − 1
24. (? Punkte)
Beweisen Sie mittels vollständiger Induktion:
n
X
i=1
i ∗ (i!) = (n + 1)! − 1
25. Binomialkoeffizient (? Punkte)
Zeigen Sie folgende Behauptung mittels vollständiger Induktion:
n X
n
= 2n
k
k=0
Tipp für diese Teilaufgabe:
A.2. ÜBUNGEN GRUNDLAGEN
53
• Verwenden Sie nicht die Beziehung
n
k
=
n!
.
k!∗(n−k)!
• Verwenden Sie nicht den Schritt ∗2, abgeleitet von der rechten Seite
der Gleichung, sondern ermitteln Sie den Schritt aus der linken Seite
der Gleichung.
•
a−1
X
j=−1
j = −1 + (
a
X
j=0
j) − a
• n, k ∈ N und k ≤ n + 1:
n
n
n+1
+
=
k−1
k
k
•
n
n
=
0
0
=1
n
−1
=
n
n+1
=0
26. Beweistechniken (? Punkte)
Beweisen Sie mittels vollständiger Induktion: Für alle positiven, natürlichen Zahlen n ≥ 24 gilt, dass sie sich durch eine Summe von 5en und 7en
darstellen lassen. Beschreiben Sie explizit, welche „Aktion“ beim Übergang
von n nach n + 1 den Schritt beschreibt.
Tipp: Unterscheiden Sie beim Induktionsschritt die Fälle, wenn in der Darstellung von n keine, genau eine, bzw. mindestens zwei 7en enthalten sind.
27. vollständige Induktion (? Punkte)
Die Teilbarkeit auf natürlichen Zahlen sei wie folgt definiert (gesprochen als
n teilt m):
n|m :⇔ ∃k ∈ N : n ∗ k = m
Beweisen Sie mittels vollständiger Induktion über die Teilbarkeit: Jede natürliche Zahl n ∈ N lässt sich als Produkt n = 2k ∗m mit natürlichen Zahlen
k und m darstellen, wobei k ∈ N0 und m ∈ N ungerade ist.
28. strukturelle Induktion (? Punkte)
Die Menge A der arithmetischen Ausdrücke über N0 sei induktiv wie folgt
definiert:
• Jede natürliche Zahl a ∈ N0 ist ein atomarer arithmetischer Ausdruck,
d.h. ∀a ∈ N0 : a ∈ A
• Seien x, y ∈ A arithmetische Ausdrücke, dann sind (x + y) und (x · y)
arithmetische Ausdrücke, d.h. ∀x, y ∈ A : (x + y), (x · y) ∈ A.
54
ANHANG A. ÜBUNGSAUFGABEN
Zeigen Sie mittels struktureller Induktion: In jedem arithmetischen Ausdruck von A ist die Anzahl von Zahlen aus N0 um 1 höher als die Anzahl
der Operatorsymbole + und ·.
29. strukturelle Induktion (? Punkte)
Für jede aussagenlogische Formel F und jede aussagenlogische Variable
P, Q, sei F [P 7→ Q] die Formel, die durch das Ersetzen aller Vorkommen
von P in F durch Q entsteht. Beispiele:
(P ∧R)[P 7→ Q] = Q∧R
((P → R)∧(P ∨Q))[P 7→ Q] = ((Q → R)∧(Q∨Q))
(R ∨ S)[P 7→ Q] = R ∨ S
P [P 7→ Q] = Q
R[P 7→ Q] = R
Beweisen Sie folgende Behauptung durch strukturelle Induktion (ähnlich
vollständiger Induktion, jedoch nicht über n, sondern über den strukturellen
Aufbau): Sei A die Interpretation (und Belegung) von Formeln der Aussagenlogik, mit A(P ) = A(Q), d.h. P und Q sind unter A semantisch
äquivalent. Dann gilt für jede Formel F : A(F ) = A(F [P 7→ Q]).
Hinweis: Sie können der Einfachheit halber statt der natürlichsprachlichen,
etwas aufwendigeren Schreibweise, bei der Interpretation A dies wie folgt
schreiben: A(¬F ) ≡ ¬A(F ) bzw. A(F1 ∧ F2 ) ≡ A(F1 ) ∧ A(F2 ). Die Induktionsbehauptung bezieht sich auf die echten Teilformeln einer Formel.
30. vollständige Induktion (? Punkte)
In einem ebenen, konvexen n-Eck (3 ≤ n ∈ N) ist die Anzahl aller Diagonalen D(n) = n∗(n−3)
.
2
31. vollständige Induktion (? Punkte)
Das linke Bild zeigt ein 4 × 4-Gitter, bei dem ein Feld entfernt wurde (mit
X gekennzeichnet), und das rechte Bild zeigt eine Abdeckung dieses Gitters
mit L-Plättchen, die durch verschiedene Buchstaben dargestellt wurden. Ein
L-Plättchen deckt drei Felder auf die angegebene Art („L“-Form)ab. Die
Buchstaben spielen keine Rolle, wichtig ist nur die L-Form. Die Größe aller
Felder ist (im Unterschied zur Darstellung) als gleich anzunehmen!
X
o o k
o e e
b X e
b b v
k
k
v
v
Zeigen Sie:
(a) Für jedes n ∈ N gilt: jedes 2n × 2n -Gitter, bei dem ein beliebiges Feld
entfernt wurde, kann mit L-Plättchen abgedeckt werden, so dass jedes
Feld von genau einem L-Plättchen abgedeckt wird.
A.2. ÜBUNGEN GRUNDLAGEN
55
n
(b) Für jedes n ∈ N gilt: Zur Abdeckung eines
2n -Gitter, bei dem ein
Pn2 ×2∗(i−1)
beliebiges Feld entfernt wurde, werden
L-Plättchen zur
i=1 2
Abdeckung benötigt, damit jedes Feld von genau einem L-Plättchen
abgedeckt wird.
32. (? Punkte)
Um nicht bei jeder Preiserhöhung des Portos durch die Post neue Marken
kaufen zu müssen, haben Sie sich überlegt, einen riesigen Vorrat an 4 und
5 Cent Marken zu kaufen. Zeigen Sie mittels vollständiger Induktion, dass
Sie jeden Betrag n ≥ 12 durch eine Kombination mit diesen 4 Cent und 5
Cent Briefmarken abdecken können.
33. vollständige Induktion (? Punkte)
Zeigen Sie: Jede natürliche Zahl 2 ≤ X ∈ N lässt sich als Produkt von
Primzahlen darstellen.
Zur Information: Eine Primzahl p ist eine natürliche Zahl (p?N), die genau
zwei unterschiedliche natürliche Zahlen als Teiler hat: die eins und sich
selbst.
34. (? Punkte)
Gegeben seien nachfolgende Ruby-Programme:
def funk1(n)
if(n < 1) raise
’natuerliche Zahl n >=1’
end
n*(2*n + 1)
end
def funk2(n)
if(n < 1) raise
’natuerliche Zahl n >=1’
end
sum = 0
for i in 1..n do
sum = sum + (4*i-1)
end
sum
end
Beweisen Sie mittels vollständiger Induktion: funk1(n) == funk2(n). Tipp:
Sie können dazu die Programme zunächst in mathematische Formeln umwandeln.
35. (? Punkte)
Gegeben seien nachfolgende Ruby-Programme:
def funk1(n)
if (n < 1) raise
’natuerliche Zahl n >= 1’
def funk2(n)
if( n < 1) raise
’natuerliche Zahl n >= 1’
56
ANHANG A. ÜBUNGSAUFGABEN
end
(n *((n*n)+2))
end
end
n == 1 ? 3 :
funk2(n-1)+3*((n-1)*n+1)
end
Beweisen Sie mittels vollständiger Induktion: funk1(n) == funk2(n).
Tipp: Wandeln Sie die Programme dazu in mathematische Formeln um.
36. Gegeben sei folgende Matrix:

1 1 0
A= 0 1 1 
0 0 1

Zeigen Sie folgende Behauptung mittels vollständiger Induktion:

1 n
n

A=A =
0 1
i=1
0 0
n
Y
n∗(n−1)
2
n
1


37. (? Punkte)
Für alle n ∈ N ist die Summe der ersten n ungeraden, natürlichen Zahlen
gleich n2 .
38. (? Punkte)
Für alle n ∈ N ist die Summe der ersten n geraden, natürlichen Zahlen
gleich der Summe der n-ten Quadratzahl mit der Zahl n selbst.
39. (? Punkte)
Beweisen Sie mittels vollständiger Induktion (ohne Transformation der Formeln!): Für alle geraden, natürlichen Zahlen n gilt
n
n
2 + 4 + 6 + . . . + n = ( )2 + .
2
2
40. Beweisfehler (? Punkte)
Überprüfen Sie die Gültigkeit der nachfolgenden Aussage und finden Sie den
Fehler in nachfolgendem Induktionsbeweis. Hinweis: der Fehler im Induktionsbeweis ist nicht gefunden, wenn man die Ungültigkeit der Aussage an
sich zeigt!
Behauptung: Seien a, b ∈ N mit max(a, b) = n, so folgt a = b.
Beweis durch vollständige Induktion:
A.2. ÜBUNGEN GRUNDLAGEN
57
Induktionsanfang Für n = 1: Wenn wir max(a, b) = 1 mit beliebigen
a, b ∈ N haben, dann können a und b wegen a, b ∈ N nur beide 1 sein.
Also gilt a = b.
Induktionsbehauptung Die Behauptung sei für ein beliebiges n gezeigt.
Induktionsschritt Ist nun max(a, b) = n + 1, dann gilt
n = max(a, b) − 1 = max(a − 1, b − 1)
Nach Induktionsbehauptung gilt also a − 1 = b − 1 und deshalb a = b.
41. Beweisfehler (? Punkte)
Gegeben sei nachfolgender Beweis. Irgendetwas ist jedoch faul daran. Nur
was ? Es ist hier nicht nach einem Gegenbeweis der Behauptung gefragt,
sondern nach dem „Denkfehler“ im induktiven Beweis.
Satz A.1 Sei n ≥ 2 und seien l1 , l2 , . . . , ln verschiedene Geraden in der
Ebene (also z.B. Geraden auf der (unendlich grossen) Tafel in der Vorlesung). Keine zwei Geraden sind parallel zueinander. Dann haben alle diese
Geraden einen gemeinsamen Punkt in der Ebene.
Beweis Durch vollständige Induktion über n:
Induktionsanfang Sei n = 2. Dann schneiden sich zwei verschiedene, nicht parallele Geraden in genau einem Punkt. Durch eine
Zeichnung ist dies leicht nachvollziehbar.
Induktionsbehauptung Die Behauptung sei für ein n gezeigt.
Induktionsschritt Seien nun l1 , . . . , ln , ln+1 Geraden mit den geforderten Eigenschaften gegeben. Nach Induktionsvoraussetzung haben die ersten n-Geraden l1 , . . . , ln−2 , ln−1 , ln einen gemeinsamen
Punkt, den wir x1 nennen. Genauso haben aber auch die n-Geraden
l1 , , . . . , ln−2 , ln−1 , ln+1 einen gemeinsamen Punkt, den wir x2 nennen. Die Gerade l1 liegt in beiden Gruppen und enthält also die
beiden Punkte x1 und x2 . Die Gerade ln−1 liegt aber auch in beiden
Gruppen und hat auch diese beiden Punkte. Da aber zwei Geraden
unter den geforderten Eigenschaften sich nur in einem (eindeutigen) Punkt schneiden können, muß x1 = x2 gelten.
42. Beweisfehler (? Punkte)
Überprüfen Sie die Gültigkeit der nachfolgenden Aussage und finden Sie den
Fehler in nachfolgendem Induktionsbeweis.
Behauptung: Es passen unendlich viele Sandkörner auf die Ladefläche eines
LKWs. Beweis durch vollständige Induktion:
58
ANHANG A. ÜBUNGSAUFGABEN
Induktionsanfang Für n = 1: da ein Sandkorn sehr klein ist, passt auf
jeden Fall ein Sandkorn auf die Ladefläche eines LKWs.
Induktionsbehauptung Die Behauptung sei für ein beliebiges n gezeigt.
Induktionsschritt Gehen wir davon aus, dass n Sandkörner auf der Ladefläche des LKWs sind. Da ein Sandkorn sehr, sehr klein ist im Vergleich zur Laderfläche eines LKWs, passt ein zusätzliches Sandkorn auf
jeden Fall auch auf die Ladefläche des LKWs. (Schritt: ein Sandkorn
auf die Ladefläche des LKWs legen.) Damit passen auch n + 1 Sandkörner auf die Ladefläche des LKWs. Daraus folgt, es passen beliebig
viele Sandkörner auf die Ladefläche eines LKWs.
43. Beweisfehler (? Punkte)
Finden Sie den Fehler in folgendem Beweis:
Behauptung: Alles was nicht rot ist, ist blau.
Beweis: Wir werden die Behauptung durch Widerspruch beweisen. Nehmen
wir also die Negation der Behauptung an: Alles was rot ist, ist blau. Dies ist
aber ein Widerspruch zur Eindeutigkeit der Farbe. Also kann die Annahme
nicht gelten und die Behauptung ist bewiesen.
44. Beweisfehler (? Punkte)
Finden Sie den Fehler in folgendem Beweis:
Behauptung: In einer Gruppe von Tieren, in der ein Elefant ist, sind alle
Tiere Elefanten.
Beweis: Wir werden die Behauptung mittels vollständiger Induktion über
die Anzahl n der Tiere in der Gruppe beweisen.
Induktionsanfang n = 1: Eine Gruppe von einem Tier in der ein Elefant
ist, besteht nur aus genau einem Elefanten, also sind alle Tiere in der
Gruppe Elefanten.
Induktionsbehauptung Die Behauptung ist für Gruppen mit n Tieren
(n beliebig) bewiesen.
Induktionsschritt Wir haben nun eine Gruppe mit n + 1 Tieren in der
ein Elefant ist. Nun nehmen wir ein Tier aus der Gruppe, so dass noch
mindestens ein Elefant in der verbleibenden Gruppe ist. Also besteht
die verbleibende Gruppe aus n Tieren, von denen eines ein Elefant ist.
Nach Induktionsbehauptung besteht diese Gruppe nur aus Elefanten.
Nun tun wir das vorher weggenommene Tier wieder hinzu und entfernen dafür analog ein anderes Tier. Wieder erhalten wir eine Gruppe
mit n Tieren von denen mindestens eines ein Elefant ist. Also sind
auch hier alle Tiere Elefanten. Dies kann nun beliebig oft wiederholt
werden. Insgesamt sind also alle Tiere Elefanten.
A.2. ÜBUNGEN GRUNDLAGEN
59
45. (? Punkte)
Eine Gerade unterteilt eine Ebene in zwei Gebiete. Zeigen Sie, dass man
2
eine Ebene mit n ∈ N0 Geraden maximal in n +n+2
Gebiete unterteilen
2
kann.
46. (? Punkte)
Eine n-elementige Menge M besitzt stets 2n Teilmengen, d.h. |P (M )| = 2n ,
also die Potenzmenge beinhaltet 2n Elemente. Weisen Sie die Korrektheit
mittels vollständiger Induktion nach.
47. (? Punkte)
Zeigen Sie mittels vollständiger Induktion: n-unterschiedliche Elemente lassen sich auf genau n! unterschiedliche Möglichkeiten anordnen.
Aufgabe A.8 (Modulare Arithmetik)
1. (? Punkte)
Lösen Sie „von Hand“, d.h. geschickt: (59 + 95) mod 8.
2. (? Punkte)
Lösen Sie „von Hand“, d.h. geschickt: (59 ∗ 95) mod 8.
3. (? Punkte)
Lösen Sie „von Hand“, d.h. geschickt: 316 mod 17.
4. (? Punkte)
Lösen Sie „von Hand“, d.h. geschickt: 310000 mod 85.
5. (? Punkte)
Verwenden Sie modulare Arithmetik, um die folgenden Aussagen zu beweisen:
(a) 1234569 ∗ 90129(mod10) ≡ 1(mod10)
(b) 2468 ∗ 13579(mod25) ≡ −3(mod25)
6. (? Punkte)
Sei x eine Variable. Bestimmen Sie eine Lösung für die Kongruenz
(7 ∗ x = 3) mod 23.
7. (? Punkte)
Bestimmen Sie die kleinste natürliche Zahl x ∈ N, für die x mod 6 = 4 und
x mod 7 = 2 gilt. Welches ist die zweit kleinste natürliche Zahl mit diesen
Eigenschaften?
8. Euklidischer Algorithmus (? Punkte)
Benutzen Sie den Euklidischen Algorithmus (siehe Tipp) zur Berechnung
von:
60
ANHANG A. ÜBUNGSAUFGABEN
(a) ggT (55, 34)
(b) ggT (960, 2304)
9. Kleiner Fermat’sche Satz (? Punkte)
Der kleine Fermat’sche Satz lautet:
Satz A.2 Sei a, p ∈ N, p eine Primzahl. Dann gilt: ap ≡ a(mod p).
Wenn a kein vielfaches von p ist, gilt: ap−1 ≡ 1(mod p).
Berechnen Sie mit Hilfe dieses Satzes:
(a) 2812 mod 13
(b) 315 mod 13
(c) 1583 mod 13
10. Teilbarkeitsregeln (? Punkte)
Es sei n eine
natürliche Zahl mit der Dezimaldarstellung ak ak−1 . . . a1 a0 ,
Pbeliebige
k
j
d.h. n =
j=0 aj ∗ 10 (Beispiel: n = 15, dann a1 = 1, a0 = 5 bzw.
1
0
15 = 1 ∗ 10 + 5 ∗ 10 ).
(a) Begründen Sie, warum eine natürliche Zahl genau dann teilbar durch
2 ist, wenn ihre Endziffer (a0 ) im Dezimalsystem durch 2 teilbar ist.
Tipp: Betrachten Sie dazu die Dezimaldarstellung bzw. Berechnung einer beliebigen natürlichen Zahl, die auf jeden Fall durch 2 teilbar ist.
(b) P
Begründen Sie, warum 9 die Zahl n teilt, wenn 9 die Quersumme
k
j=0 aj von n teilt. Tipp: Betrachten Sie dazu die Dezimaldarstellung bzw. Berechnung einer beliebigen natürlichen Zahl, die auf jeden
Fall durch 9 teilbar ist.
(c) Was ist die Endziffer (a0 ) der Dezimaldarstellung von 51n ? (Begründung nicht vergessen)
A.3. ÜBUNGEN KOMBINATORIK
A.3
61
Übungen Kombinatorik
Aufgabe A.9
1. Wieviele verschiedene Anordnungen der Buchstaben des Wortes “statistics“ gibt es
2. Wieviele Möglichkeiten gibt es, 10 Personen auf eine Bank mit nur 4 Plätzen zu setzen ?
3. Wieviele Möglichkeiten an Tipps gibt es im Zahlenlotto “6 aus 49“ ?
4. Wieviele vierstellige Zahlen können mit den Ziffern {0, 1, 2, 3, 4, 5, 6, 7, 8, 9}
gebildet werden:
(a) mit Wiederholung ?
(b) ohne Wiederholung ?
(c) ohne Wiederholung und die letzte Ziffer muß die Null sein ?
Beachten Sie dabei, dass normalerweise bei den vierstelligen Zahlen diejenigen mit führenden Nullen (z.B. 042, 001, ...) nicht mitgezählt werden!
5. Auf wieviele Arten können 10 Schüler einer Klasse auf Sitzplätzen angeordnet werden,
(a) wenn genau 10 Sitzplätze vorhanden sind ?
(b) wenn 12 Sitzplätze vorhanden sind ?
6. An einer europäischen Konferenz nehmen zwei Deutsche, vier Engländer,
drei Franzosen und zwei Niederländer teil. Auf wieviele Arten können sie
(a) an einem U-förmigen Tisch
(b) an einem runden Tisch
angeordnet werden, wenn die Personen der gleichen Nationalität zusammensitzen sollen ?
Hinweis: An den „Innenseiten“ der Tische sitzt niemand.
7. Fußball WM (? Punkte)
Bei der Fußball-WM 1998 nahmen 32 Nationen teil. Wie viele Möglichkeiten
gab es
(a) für die Teilnehmer des Halbfinales (,d.h. Runde der letzten vier) ?
(b) für die Reihenfolge auf den ersten vier Plätzen ?
Vereinfachen Sie jeweils den arithmetischen Ausdruck soweit, dass kein
Bruch mehr darin vorkommt.
62
ANHANG A. ÜBUNGSAUFGABEN
Aufgabe A.10 In der Sendung mit Thomas Gottschalk am 26 Februar 2000
gab es folgende Wette: Auf einem Sideboard standen 20 Schreibtischlampen, wovon jedoch nur 4 am Stromnetz angeschlossen waren. Dies war von außen nicht
sichtbar, da die Glühlampen leicht herausgedreht waren. Der „Wettende“ behauptete, dass er mit bloßen Händen fühlen könne, in welchen der Lampen Strom
fließe. Bei der Wette wollte er zuerst alle 20 Lampen anfassen, dann 4 davon als
„am Netz“ benennen. Dabei bestand die Wette in der Sendung konkret darin, dass
er mindestens drei der 4 angeschlossenen Lampen richtig benennt. Der Literaturkritiker Marcel Reich-Ranicki war „Wettepate“ und sagte: „Ich glaube nicht an
Wunder, also auch nicht an diese Wette!“ Trotzdem: Die Wette wurde gewonnen,
drei der vier Lampen, die der Wettende benannte, waren am Netz!
Gesucht:
• Die Anzahl der Möglichkeiten vier Lampen aus 20 auszuwählen.
• Die Anzahl der Möglichkeiten drei Lampen aus 20 auszuwählen.
Aufgabe A.11 (Glasperlen) (? Punkte)
Eine Firma, die Glasperlen herstellt, will testen, welche Farben für Kinder besonders attraktiv sind. In 6 Glasschüsseln befinden sich jeweils 12 Glasperlen
einer ganz bestimmten Farbe. In der Glasschüssel 1 sind z.B. 12 rote Perlen. Das
Kind wird vor die Glasschüsseln gebracht und erhält die Anweisung: Du darfst
dir 10 Perlen auswählen und mit nach Hause nehmen. Für das Experiment entscheidend ist die Frage, wie oft das Kind welche Farbe ausgewählt hat. Wie viele
unterschiedliche Auswahlmöglichkeiten gibt es ?
Aufgabe A.12 Den Studierenden stehen im Fachbereichsrat 3 Sitze zu. Um diese Sitze konkurrieren insgesamt 6 studentische Verbände A, B, C, D, E und F.
Jeder Verband verfügt jeweils über mindestens 3 Bewerber, d.h. jeder Verband
hat genügend Bewerber, um alle Sitze zu belegen, da jeder Verband mehrere Sitze
erhalten kann. Wie viele Sitzverteilungen unter den Verbänden sind möglich ?
Die einzelnen Bewerber eines Verbandes werden hier nicht unterschieden.
Aufgabe A.13 Ein Teil der Belegschaft von Apollo 13 besteht üblicherweise aus
einem Pilot, einem Copilot und einem Geologen. Die NASA hat neun Flieger und
vier Geologen als Kandidaten für den Flug ausgebildet. Wie viele verschiedene
Belegschaften können zusammen gestellt werden?
Aufgabe A.14 Ein moderner Komponist mit Hang zur aleatorischen Computermusik erhält den Auftrag, zu Ehren des großen Komponisten Bach ein neues
Werk zu komponieren. Die bestechende Konstruktionstechnik sieht wie folgt aus:
Ein Takt bestehe aus 4 aufeinanderfolgenden Tönen. Die möglichen Töne sind „a
b c h“. Innerhalb eines Taktes kann ein bestimmter Ton beliebig oft vorkommen.
Das Kompositionswerk besteht aus einer zufälligen Anordnung aller möglichen
A.3. ÜBUNGEN KOMBINATORIK
63
Takte und endet dann, wenn die Folge „b a c h“ auftritt. Wieviele unterschiedliche Takte gibt es, (d.h. wieviele Takte muß der Zuhörer im ungünstigsten Fall
über sich ergehen lassen, bis das Werk beendet ist, wenn jeder Takt nur einmal
auftreten darf ?)
Aufgabe A.15 Ein moderner Künstler erhält vom Bundespräsidenten den Auftrag, für den Geburtstag der Bundesrepublik Deutschland ein Bild zu malen. Der
Künstler hat sich bereits darauf festgelegt, dass das Bild aus 12 nebeneinander angeordneten Streifen (hohe schmale Rechtecke) bestehen soll und 4 Streifen
schwarz, 4 Streifen rot und 4 Streifen gold sein müssen. Die eigentliche künstlerische Herausforderung besteht darin, die Farben optimal auf die Streifen zu
verteilen. Unter wievielen Möglichkeiten kann der Künstler auswählen ?
Aufgabe A.16 Auf einer CD sind 9 verschiedene Musikstücke. Sie tippen auf
die Shuffle-Taste, welche alle Titel in zufälliger Reihenfolge anbietet. Wieviele
mögliche Reihenfolgen gibt es insgesamt?
Aufgabe A.17 Bitte beachten Sie, dass Sie bei den Lösungen zunächst klären
müssen, um welchen Fall es sich in der Tabelle handelt und warum, etwa in der
Art „Es handlet sich hier um Kombinationen zur Klasse/Ordnung eins ohne Wiederholung, weil ...“. Die reine Angabe der Lösung oder der Formel zur Berechnung
der Lösung genügt nicht!
1. Zu Weihnachten werden im Vorfeld der Veranstaltung 16 Taschen auf den
16 Sitzplätzen verteilt. Bei 14 Taschen ist der Inhalt gleich. Wieviele unterscheidbare Permutationen gibt es ?
2. Wieviele Zeichen sind in einem Byte (= 8 Bit) darstellbar ?
3. Beim Zahlenlotto „6 aus 49“ werden in jedem Feld sechs Zahlen angekreuzt.
Wie groß ist die Wahrscheinlichkeit, mit einem Feld drei Richtige bei einer
Ziehung zu tippen ?
4. Eine Urne enthält vier schwarze und sechs weiße Kugeln. Es werden drei
Kugeln gleichzeitig gezogen (ohne Zurücklegen). Wieviele Möglichkeiten gibt
es, mehr weiße als schwarze Kugeln zu ziehen ?
5. Sie haben 17 (paarweise ungleiche) natürliche Zahlen ni , von denen keine
einen Primfaktor größer als 7 hat. (Primfaktoren: 2, 3, 5, 7). Zeigen Sie,
dass wenigstens zwei Zahlen ni , nj , i 6= j darunter sind, deren Produkt eine
Quadratzahl ist.
Hinweis: jede natürliche Zahl lässt sich (bis auf die Reihenfolge) eindeutig
als Produkt von Primzahlen darstellen. Für eine Quadratzahl n gilt: ∃k ∈
N : n = k2.
64
ANHANG A. ÜBUNGSAUFGABEN
Aufgabe A.18
1. Sie haben den Auftrag, aus 9 verschiedenen Bewerbern 3
auszuwählen und diese 3 in der Rangreihe ihrer Eignung vorzuschlagen.
Wieviele Möglichkeiten gibt es ?
2. Sie planen in einem Weinseminar die Qualität von 5 Weinen zu testen.
Dabei soll für alle möglichen Weinpaare getestet werden, welcher von den 2
Weinen der bessere ist. Ein Weinkenner behauptet nun, es käme sehr darauf
an, welcher Wein beim Paarvergleich zuerst gekostet wird. Sie legen deshalb
fest, dass bei jedem möglichen Paarvergleich auch die Reihenfolge getestet
werden muss, also z.B. einmal die Reihenfolge Wein A Wein B sowie auch
die Reihenfolge Wein B Wein A. Wieviele Vergleiche sind notwendig, um
alle Weine in dieser Art gegeneinander zu testen?
3. Modellierung (?? Punkte)
Sie planen in einem Schokoladen-Seminar die Qualität von fünf Schokoladenkreationen zu testen. Dabei soll für alle möglichen Paare getestet werden, welcher von den jeweils zwei Schokoladenkreationen die bessere ist. Ein
Schokoladenkenner behauptet nun, es käme sehr darauf an, welche Schokoladenkreation beim Paarvergleich zuerst gekostet wird. Sie legen deshalb fest,
dass bei jedem möglichen Paarvergleich auch die Reihenfolge getestet werden
muss. Wieviele Vergleiche sind notwendig, um alle Schokoladenkreationen
in dieser Art gegeneinander zu testen ?
4. Ein Entwicklungspsychologe will herausfinden, ob ein Kind den Größenbegriff versteht. Dazu legt er dem Kind nach Zufall 6 unterschiedlich große
Figuren vor und fordert es auf: „Stell die Figuren auf den Tisch und ordne
Sie nach ihrer Höhe!“ Wie viele Möglichkeiten gibt es, die Figuren der Höhe
nach in eine Reihenfolge zu bringen ?
5. An einer Feier nehmen 20 Personen teil. Plötzlich geht das Bier aus. Um
hinreichenden Nachschub zu besorgen, werden 3 Leute ausgewählt, weil 3
Personen notwendig sind, um das neue Fass zu transportieren. Wieviele
unterschiedliche Möglichkeiten gibt es, 3 Leute zum Bierholen zu schicken ?
6. Aufgaben (? Punkte)
In einer Mathematiklausur werden 6 Aufgaben aus der Logik, 4 Aufgaben
aus den Grundlagen und 5 Aufgaben aus der Stochastik gestellt. Die Studierenden dürfen sich je 2 Aufgaben aus jedem Stoffgebiet aussuchen. Wie
viele verschiedene Möglichkeiten der Zusammenstellung haben sie ?
7. Kombination (? Punkte)
Ein Gärtner pflanzt 10 Zwiebeln einer rot blühenden Tulpensorte und 10
Zwiebeln einer gelb blühenden in zwei Reihen mit acht und 12 Zwiebeln.
Wie viele Möglichkeiten der Bepflanzung gibt es, wenn sich in einer der
Reihen genau vier Zwiebeln der gelb blühenden Tulpensorte befinden sollen?
A.3. ÜBUNGEN KOMBINATORIK
65
Vereinfachen Sie den arithmetischen Ausdruck soweit, dass kein Bruch
mehr darin vorkommt.
8. Begrüßung (? Punkte)
(a) 12 Personen begrüßen sich jeweils mit Hände schütteln. Wie oft werden
Hände geschüttelt ?
(b) 12 Personen begrüßen sich jeweils mit Küsschen links, Küsschen rechts,
Küsschen links. Wie viele Küsschen gibt das ?
Die arithmetischen Ausdrücke sind zu kürzen, bis kein Bruch mehr notwendig ist.
9. Modellierung (?? Punkte)
Lena arbeitet bei einer Firma, die Großveranstaltungen ausrichtet, und bekommt gelegentlich 5 Freikarten (ihre eigene Karte nicht mitgerechnet), die
sie an ihre 10 guten Bekannten verteilt. Um niemanden zu benachteiligen,
verlost sie die Karten.
(a) Wie viele Möglichkeiten der Verteilung gibt es ?
(b) Kurz vor der nächsten Veranstaltung haben sich zwei ihrer Bekannten
frisch verliebt und sind absolut unzertrennlich. Wie viele Möglichkeiten gibt es, wenn die beiden nur zusammen oder gar nicht kommen
wollen ?
(c) Viele Monate später gibt es Nachwuchs. Da das Paar den Säugling zu
Beginn keinem Babysitter anvertrauen möchte, kann höchstens einer
der beiden mitkommen. Wie viele Möglichkeiten der Verteilung hat
Lena nun ?
10. Wahlen (? Punkte)
(a) Bei einer Wahl geben 101 Personen jeweils eine Stimme für einen von
vier Kandidaten ab. Wie viele mögliche Wahlausgänge gibt es ?
(b) Bei einer Wahl geben 101 Personen jeweils eine Stimme für einen
von drei Kandidaten ab. Wie viele mögliche Wahlausgänge gibt es, bei
denen keiner der Kandidaten mehr als 50 Stimmen erhält ?
Hinweis: Hierbei ist die Lösung von (a) nicht von Nutzen. Betrachten Sie stattdessen, wie viele Stimmen der zweite Kandidat bekommen
kann, falls der erste i Stimmen erhält.
11. Wahlen (? Punkte)
66
ANHANG A. ÜBUNGSAUFGABEN
(a) In einer Mathematikarbeit werden 6 Aufgaben aus den Grundlagen, 4
Aufgaben aus der Beweisführung und 5 Aufgaben aus der Kombinatorik
gestellt. Die Schüler dürfen sich je 2 Aufgaben aus jedem Stoffgebiet
aussuchen. Wie viele verschiedene Möglichkeiten der Zusammenstellung haben sie ?
(b) Eine Reisegruppe von 12 Personen verteilt sich auf 2 Abteile eines Eisenbahnwagens. In jedem Abteil gibt es 3 Sitzplätze in Fahrtrichtung
und 3 Sitzplätze gegen die Fahrtrichtung. Von den 12 Personen wollen
auf alle Fälle 5 in Fahrtrichtung und 4 gegen die Fahrtrichtung sitzen. Wie viele Plazierungsmöglichkeiten gibt es, wenn man die Sitze
unterscheidet ?
Die arithmetischen Ausdrücke sind jeweils zu kürzen, bis kein Bruch mehr
notwendig ist.
12. Vereinsmeierei (? Punkte)
Der Deutsche Kaninchenzüchterverein e.V. beschäftigt sich wieder mit sich
selbst. Ein neuer Vorstand soll gewählt werden, bestehend aus dem 1. Vorsitzenden, 2. Vorsitzenden, Kassenwart, Revisor und Pressesprecher. Das
bedeutet, dass jedes zweite Mitglied ein Amt erhält, denn der Verein umfasst
nur 10 Mitglieder.
Die Igelzüchter 1902 e.V. finden das sehr spießig. Sie haben selbstverständlich keinen Vorstand. Allerdings wird auch dort gewählt: Um effektiv
arbeiten zu können, benötigen die 10 Stacheltierfreunde einen fünfköpfigen
geschäftsführenden Ausschuss.
Wie viele unterschiedliche Ergebnisse kann die Wahl zum
(a) Vorstand des Deutsche Kaninchenzüchterverein e.V.
(b) Geschäftsführenden Ausschuss der Igelzüchter 1902 e.V.
erbringen?
Die arithmetischen Ausdrücke sind zu kürzen, bis kein Bruch mehr notwendig ist.
13. Ein Skatspiel besteht aus 32 unterschiedlichen Karten. Ein Hellseher gibt
die Anweisung: „A: Alle Karten mischen; eine Karte entnehmen, notieren
und zurücklegen. B: Prozedur A insgesamt 3 mal realisieren.“ Er behauptet nun: „Ich weiss zwar nicht, welche Karte sie beim ersten, zweiten oder
dritten mal gezogen haben, aber ich kann ihnen nachher sagen, welche Karten sie notiert haben.“ Unter wie vielen Möglichkeiten muß der Hellseher
auswählen ?
A.3. ÜBUNGEN KOMBINATORIK
67
14. Beim Fussballtotto (Elferwette) muss man die Ergebnisse aus 11 Fussballspielen vorhersagen. Wie das Ergebnis getippt werden muss, wird hier beispielhaft für ein Spiel erklärt.
Spiel Bayern München vs. Borussia Dortmund
1 = Bayern München gewinnt
2 = Borussia Dortmund gewinnt
0 = unentschieden.
Wieviele mögliche unterschiedliche Tippergebnisse gibt es ?
15. Passwort (? Punkte)
Der Zugang zu einem Computer eines Netzwerks ist durch ein zehnstelliges
Codewort gesichert. Jede Stelle des Codewortes wird aus einem Vorrat von
64 Zeichen besetzt.
(a) Wie viele unterschiedliche Passwörter können erzeugt werden ?
(b) Wie viele Möglichkeiten gibt es, ein Codewort aus lauter verschiedenen
Zeichen zu erstellen ?
(c) Wie viele Möglichkeiten gibt es, ein Codewort zu erstellen, dass nur
aus Zeichen einer festen Teilmenge von 30 Zeichen besteht ?
16. Buffet (? Punkte)
Acht Personen warten vor dem Selbstbedienungsbuffet.
(a) Auf wie viele Arten kann die Schlange zusammengesetzt sein ?
(b) Drei der acht Personen wählen das Fischgericht. Wie viele Möglichkeiten gibt es für die Auswahl dieser drei Personen?
(c) Die drei Fischliebhaber stehen direkt hintereinander. Wie viele unterschiedliche Schlangen sind möglich?
17. Permutation (? Punkte)
A := {a, b, c, d, e, f } sei ein Alphabet. Als P bezeichnen wir die Menge der
Permutationen (ohne Wiederholung/ohne Gleichen) des Alphabets A. Wir
bilden verschiedene Teilmengen von P und fragen nach der Anzahl ihrer
Elemente:
(a) Menge P , alle Permutationen;
(b) P1 : alle Permutationen, die den Buchstaben c auf der 2-ten Stelle enthalten;
(c) P2 : alle Permutationen, die die Kette cda enthalten;
68
ANHANG A. ÜBUNGSAUFGABEN
(d) P3 : alle Permutationen, die die Kette cda und gleichzeitig die Kette
ebf enthalten;
(e) P4 : alle Permutationen, die die Kette cda oder die Kette ebf enthalten.
Wie viele Elemente haben die Mengen P, P1 , P2 , P und P4 ? Bestimmen Sie
die Anzahl durch Berechnung und nicht durch explizite Angabe!
18. Zeit (?? Punkte)
Sisyphos bekommt den Auftrag, alle Anordnungen von zwölf Werken der
griechischen Antike aufzustellen. Er schafft pro Minute eine Anordnung
und arbeitet 16 Stunden am Tag. Wie viele Jahre (á 365 Tage) benötigt er
für die Arbeit? Kürzen Sie den einstehenden Ausdruck soweit wie möglich.
19. Primzahlen (? Punkte)
Zur Europameisterschaft im Fußball hat die Firmenchefin der Schokoladenfabrik „Lecker 2000“ als Hobbymathematikerin ein Preisrätsel veranstaltet.
Beantworten Sie die beiden dort gestellten Fragen mittels Kombinatorik:
(a) Auf wie viele Arten kann man die Primfaktoren in der Primfaktordarstellung der Zahl 2000 anordnen ?
(b) Wie viele verschiedene Teiler hat die Zahl 2000 ?
20. Bauteile (? Punkte)
Ein Autoteilezulieferer stellt für eine Autofirma ein aufwendiges elektronisches Bauteil her. In einem Karton befinden sich 50 unterscheidbare Bauteile, von denen genau vier fehlerhaft sind. Jemand nimmt aus dem Karton
zufällig fünf Bauteile.
(a) Berechnen Sie, wie viele Möglichkeiten es gibt, dass sich unter den fünf
herausgenommenen Bauteilen genau zwei fehlerhafte befinden.
(b) Berechnen Sie, wie viele Möglichkeiten es gibt, dass sich unter den fünf
herausgenommenen Bauteilen mindestens ein fehlerhaftes befindet.
(c) Die 50 Bauteile werden aus dem Karton genommen und nebeneinander
gelegt. Berechnen Sie, wie viele Möglichkeiten es gibt, dass die vier
fehlerhaften Bauteile genau nebeneinander liegen.
21. (? Punkte)
Die Klassen A und B sind im Ferienlager: A hat 12 Mädchen und 9 Jungen
und B hat 8 Mädchen und 16 Jungen. Die Formeln sind soweit zu vereinfachen, dass kein Bruch mehr vorhanden ist!
(a) Eine Dreiergruppe muss einkaufen gehen; wie viele Möglichkeiten zur
Bestimmung dieser Gruppe gibt es?
A.3. ÜBUNGEN KOMBINATORIK
69
(b) Eine Dreiergruppe muss einkaufen gehen; wie viele Möglichkeiten gibt
es, wenn die Gruppe nicht nur aus Mädchen oder nur aus Jungen
bestehen soll?
(c) Gesucht werden vier Kinder, die ganz verschiedene Aufträge ausführen
sollen; wie viele Möglichkeiten gibt es?
(d) Wie viele Zweiergruppen Mädchen/Junge (als Paar) sind möglich?
(e) Wie viele Zweiergruppen Mädchen/Junge (als Paar) sind möglich, wenn
die beiden aus verschiedenen Klassen kommen sollen?
(f ) Peter wurde im Zimmer beim Rauchen erwischt. Zur Strafe muss er in
lexikographischer Reihenfolge alle dreibuchstabigen „Wörter“ aufschreiben, die man aus den Buchstaben R, A, U, C und H bilden kann. (Ein
Buchstabe darf mehrmals verwendet werden.) Er hat jetzt gerade das
Wort „AAA“ geschrieben. Wie viele Wörter muss er noch schreiben?
22. Festessen (? Punkte)
12 Personen, davon sechs Damen und sechs Herren, wollen an einem runden Tisch mit 6 Plätzen Platz nehmen.
(a) Wie viele Sitzordnungen3 gibt es ?
(b) Bei wie vielen Sitzordnungen sitzen nur Damen am Tisch ?
(c) Bei wievielen Sitzordnungen sitzen Damen und Herren abwechselnd
am Tisch ?
(d) Bei wievielen Sitzordnungen sitzen 4 Damen und 2 Herren am Tisch ?
(e) Bei wievielen Sitzordnungen sitzen 4 Damen und 2 Herren so am
Tisch, dass die beiden Herren nicht nebeneinander sitzen ?
23. Urlaub (? Punkte)
Acht Briten (vier Ehepaare) machen Urlaub auf Madeira.
(a) Sie haben vier Doppelzimmer gebucht (pro Paar eines). Wie viele Verteilungsmöglichkeiten gibt es ?
(b) Am Pool stehen 12 Liegestühle zur Verfügung. Auf wie viele Arten
können sie belegt werden ?
(c) Für das Erinnerungsfoto besteht der Fotograf darauf, dass Frauen und
Männer abwechselnd in einer Reihe sitzen. Wie viele Anordnungsmöglichkeiten gibt es ?
(d) Täglich gibt es die Auswahl aus drei Mittagsmenüs, wobei sich der
Speiseplan nach einer Woche wiederholt. Wie viele Speiseauswahlmöglichkeiten hat jeder Brite,
3
Unter einer Sitzordnung versteht man eine feste Zuordnung von 6 der 12 Personen zu den
6 Plätzen!
70
ANHANG A. ÜBUNGSAUFGABEN
• wenn der Urlaub sieben Tage dauert?
• der Urlaub zehn Tage dauert und er kein Gericht zweimal essen
möchte ?
24. Modellierung (?? Punkte)
Wir betrachten das Wort MORGENS und seine Buchstaben.
(a) Bestimmen Sie die Anzahl aller Worte mit vier Buchstaben, die man
aus den Buchstaben des Wortes MORGENS bilden kann.
(b) Wie viele von den in Teil (a) gefundenen Wörter enthalten nur Konsonanten ?
(c) Wie viele von den in Teil (a) gefundenen Wörter beginnen und enden
mit einem Konsonanten ?
(d) Wie viele von den in Teil (a) gefundenen Wörter beginnen mit einem
Vokal ?
(e) Wie viele von den in Teil (a) gefundenen Wörter enthalten den Buchstaben S ?
(f ) Wie viele von den in Teil (a) gefundenen Wörter beginnen mit G und
enden mit einem Vokal ?
(g) Wie viele von den in Teil (a) gefundenen Wörter beginnen mit G und
enthalten ein R ?
(h) Wie viele von den in Teil (a) gefundenen Wörter enthalten beide Vokale ?
25. Geburtstagsparty (? Punkte) Es findet eine Geburtstagsparty mit 5 Mädchen und 3 Jungen statt. Jedes Kind erhält ein Stück Kuchen. Es stehen
5 Sorten zur Wahl: Tirolercake, Schoggicake, Marmorcake, Zitronencake,
Plumcake.
Berechnen Sie für jede beschriebene Situation die jeweilige Anzahl der Möglichkeiten. Die Teilaufgaben sind alle voneinander unabhängig!
(a) Die Kinder stehen Schlange vor dem Buffet.
Anzahl der möglichen Schlangen ?
(b) Die Jungen stehen ganz vorne in der Schlange.
Anzahl der möglichen Schlangen ?
(c) Lisa, Bea und Anna müssen immer die gleiche Sorte haben.
Anzahl der möglichen Wahlausgänge ?
(d) Jedes Kind in der Reihe wählt grundsätzlich etwas anderes als sein
Vorgänger.
Anzahl der möglichen Wahlausgänge ?
A.4. ÜBUNGEN WAHRSCHEINLICHKEITSRECHNUNG
71
(e) Es werden 3 Stück Tirolercake, 3 Stück Schoggicake und 2 Stück Marmorcake gewählt.
Anzahl der möglichen Zuordnungen zu den Kindern ?
(f ) 4 Kinder spielen „Schwarzer Peter“. Die Gruppe ist aus Jungen und
Mädchen gemischt zusammengesetzt.
Anzahl der möglichen Gruppenzusammensetzung ?
26. Arbeitsweg (? Punkte)
Die Eckpunkte der kleinen Rechtecke in nachfolgender Graphik repräsentieren Orte, wobei unten links der Punkt (0|0) ist, unten rechts der Punkt
(0|8), oben links der Punkt (8|0) und oben rechts der Punkt (8|8).
Abel wohnt in (0|0) und arbeitet in (8|8). Sein Arbeitskollege Kain wohnt in
(4|4). Abel fährt jeden Morgen zur Arbeit und nimmt unterwegs Kain mit.
Es ist vorauszusetzen, dass er am Morgen früh keine Umwege macht. Auf
wie viele Arten kann er das tun ?
27. Erleuchtung (? Punkte)
In einem Raum gibt es 8 Lampen, die man unabhängig voneinander einund ausschalten kann. Wie viele Beleuchtungsarten gibt es, wenn
(a) genau 5 Lampen brennen sollen,
(b) mindestens 5 Lampen brennen sollen?
A.4
Übungen Wahrscheinlichkeitsrechnung
Aufgabe A.19
1. Allgemein (? Punkte)
Erklären Sie
(a) Was sind Ergebnisse und Ereignisse ?
(b) Was ist eine Wahrscheinlichkeitsfunktion und wie ist sie beim LaplaceVersuch definiert ?
(c) Was ist eine Zufallsvariable und eine Verteilungsfunktion ?
72
ANHANG A. ÜBUNGSAUFGABEN
(d) Was ist der Erwartungswert und die Varianz ?
(e) Welche Rolle spielt die Wahrscheinlichkeitsrechnung innerhalb der Stochastik ?
2. Zufällige Ereignisse (? Punkte)
Vier Erzeugnisse werden geprüft. Geben Sie mit Hilfe der Ereignisse
A = { Mindestens ein Stück ist einwandfrei.} und
B = { Höchstens ein Stück ist einwandfrei.}
in Worten die Bedeutung folgender Ereignisse an:
A−1 , B −1 , A ∪ B, A ∩ B, A−1 ∩ B, A−1 ∩ B −1 und A \ B.
3. Nummernschild (? Punkte)
Meiko wettet mit Johanna: von den nächsten vier Autos, denen wir begegnen, haben mindestens zwei Autos gleiche Schlussziffern in ihrer Autonummer (gemeint ist: HH-HB 842).
Mit welcher Wahrscheinlichkeit gewinnt Meiko ?
4. Historie (? Punkte)
„Die Frage des Chevalier de Méré“: Betrachtet werden zwei Glücksspiele:
(a) Mit einem idealen Würfel wird 4-mal gewürfelt und gewettet, dass mindestens eine Sechs erscheint. Wie groß ist die Wahrscheinlichkeit dafür ?
(b) Mit zwei unterscheidbaren idealen Würfeln wird 24-mal gewürfelt und
gewettet, dass mindestens eine Doppelsechs erscheint. Wie groß ist die
Wahrscheinlichkeit dafür ?
Chevalier de Méré war ein leidenschaftlicher Glücksspieler. Seiner Meinung
nach müssten die Gewinnchancen in beiden Glücksspielen identisch sein, da
es im zweiten Spiel 6-mal mehr Elementarereignisse als im ersten Spiel gibt,
muss man auch 6 ∗ 4 = 24 Würfe erlauben. Hatte er Recht ? Begründen Sie
kurz Ihre Antwort. Wo liegt ggf. der Denkfehler ?
5. Würfel (? Punkte)
(a) Bilden Sie einen Ereignisraum Ω, der die Summe der Augenzahlen
zweier (idealer, nicht unterscheidbarer) Würfel modelliert. Geben Sie
zu den elementaren Ereignissen die Wahrscheinlichkeiten an. Begründen Sie Ihre Angabe!
(b) Betrachten Sie nun folgendes Spiel: Sie gewinnen, falls die Summe eine
der sieben Zahlen 2, 3, 5, 9, 10, 11, 12 ist, ansonsten, bei den vier Zahlen 4, 6, 7, 8, verlieren Sie. Wie groß ist die Wahrscheinlichkeit eines
Gewinns ? Begründen Sie Ihre Antwort!
A.4. ÜBUNGEN WAHRSCHEINLICHKEITSRECHNUNG
73
(c) Betrachten Sie nun drei Würfel und folgendes Spiel: Sie gewinnen,
falls eine 3 vorkommt, die Summe 15 ist oder alle drei Würfel die selbe Augenzahl haben, ansonsten verlieren Sie. Wie groß ist die Wahrscheinlichkeit eines Gewinns ?
6. Unabhängigkeit von Ereignissen (? Punkte)
In einer Urne befinden sich 10 grüne, 15 rote und 25 blaue Kugeln. Wie
groß ist die Wahrscheinlichkeit dafür, dass bei zwei aufeinander folgenden
Entnahmen zwei verschieden farbige Kugeln gezogen werden, wenn die zuerst gezogene Kugel
(a) wieder zurückgelegt und eingemischt wird,
(b) nicht wieder zurückgelegt wird ?
7. Musik CD (? Punkte)
(a) In einem Tonstudio wird eine CD mit acht Liedern und fünf Instrumentalstücken zusammengestellt. Die CD wird in einem CD-Player
mit der Random-Funktion abgespielt, so dass die 13 Musikstücke in zufälliger Reihenfolge ohne Wiederholung aufeinander folgen. Wie groß
ist die Wahrscheinlichkeit, dass unter den ersten vier gespielten Stücken
höchstens zwei Instrumentalstücke sind ?
(b) Von allen in einem Musikladen verkauften CDs entfallen 25% auf klassische Musik und 30% auf Volksmusik. Der Rest wird der Popmusik
zugeordnet. 60% der Käufer einer Klassik-CD und 25% der Käufer einer Popmusik-CD sind älter als 30 Jahre. Insgesamt werden 48% der
verkauften CDs von Kunden erworben, die älter als 30 Jahre sind.
i. Ein Kunde betritt den Musikladen und kauft eine Volksmusik-CD.
Mit welcher Wahrscheinlichkeit ist er höchstens 30 Jahre alt ?
ii. Mit welcher Wahrscheinlichkeit kauft ein Kunde, der älter als 30
Jahre ist, eine Klassik- oder Popmusik-CD ?
8. Zufallsvariable (? Punkte)
Zwei Personen gestalten folgendes Spiel: Der Spieler 1 zahlt einen bestimmten Einsatz an Spieler 2, würfelt und erhält von Spieler 2
• 20 Cent beim Würfeln einer 1 oder 2,
• 30 Cent beim Würfeln einer 3 oder 4,
• 50 Cent beim Würfeln einer 5,
• 90 Cent beim Würfeln einer 6.
74
ANHANG A. ÜBUNGSAUFGABEN
Welcher Einsatz sollte Spieler 2 mindestens verlangen, um zumindest keinen
Verlust bzgl. dem durchschnittlich zu erwartendem Gewinn zu machen ?
Tipp: Betrachten Sie die Zufallsvariable X = Einnahme des Spielers 1 pro
Spiel und berechnen den Erwartungswert E(X).
9. Modellierung
Die Intaktwahrscheinlichkeiten, bezogen auf ein festes Zeitintervall, betragen für drei unabhängig voneinander arbeitende Anlagen 0, 8 und 0, 7 und
0, 6. Die Zufallsvariable X sei die zufällige Anzahl der in einem solchen
Zeitintervall intakten Anlagen. Modellieren bzw. Bestimmen Sie
(a) Modellieren Sie für alle drei Anlagen die Ereignisse Ai := {Anlage i ist
intakt}. Bestimmen Sie ebenso P (Ai ). Begründen Sie Ihr Vorgehen.
(b) Geben Sie die Verteilungstabelle von X an, also P (X = 0), P (X =
1), P (X = 2), P (X = 3).
(c) Bestimmen Sie die Wahrscheinlichkeit dafür, das mindestens eine Anlage intakt ist.
(d) Geben Sie die Verteilungsfunktion von X an.
(e) Bestimmen Sie den Erwartungswert E(X) und die Varianz σ 2 (X).
10. Modellierung (? Punkte)
Ein Großhändler bezieht Glühlampen von drei Betrieben, und zwar 40% aus
Betrieb 1 und 50% aus Betrieb 2. Die Ausschussquoten sind 5% im Betrieb
1, 2% im Betrieb 2 und 10% im Betrieb 3. Beim Großhändler werden die
Lieferungen aus den 3 Betrieben vermischt.
Mit welcher Wahrscheinlichkeit ist eine zufällig ausgewählte Glühlampe defekt ?
11. Modellierung (? Punkte)
Im letztjährigen Skilager traten allerlei mathematische Probleme auf. Im
Speisesaal gab es 9 Tische.
(a) Der Lehrer kaufte für jeden Tisch etwas Süsses. 4 Gläser Nuss-NougatCreme, 3 Gläser Marmelade und 2 Gläser Honig. Auf wie viele Arten
konnten die Gläser verteilt werden, wenn auf jeden Tisch ein Glas
kommen sollte und Gläser von der gleichen Art nicht unterscheidbar
waren?
(b) Florian hatte am 1. Tag ein Marmeladeglas auf dem Tisch. Wie hoch
ist die Wahrscheinlichkeit, dass er am 2. Tag ebenfalls ein Marmeladeglas haben wird?
(c) Als Melanie die Tische zum Frühstücken decken musste, nahm sie zufällig vier der neun Gläser in die Hände. Wie gross ist die Wahrscheinlichkeit, dass beide Honiggläser dabei waren?
A.4. ÜBUNGEN WAHRSCHEINLICHKEITSRECHNUNG
75
(d) Ein Wetterbericht sagte, die Wahrscheinlichkeit, dass es schneien werde, sei in dieser Woche an jedem Tag 50%. Nun sind schon drei Tage
ohne Schnee vergangen. Wie hoch ist die Wahrscheinlichkeit, dass es in
den verbleibenden zwei Tagen noch an mindestens einem Tag Schnee
geben wird?
(e) Ein anderer, wissenschaftlicher Wetterbericht meinte, wenn es schneie,
dann werde der Luftdruck mit einer Wahrscheinlichkeit von 70% steigen. Andererseits, wenn es nicht schneie, dann werde der Luftdruck
mit einer Wahrscheinlichkeit von 20% nicht steigen.
Durch statistische Untersuchungen weiss man, dass es am Donnerstag
mit einer Wahrscheinlichkeit von 55% zu schneien beginnt. Mit welcher
Wahrscheinlichkeit wird am Donnerstag der Luftdruck steigen?
(f ) Am Donnerstagmorgen sieht der Lehrer, dass der Luftdruck steigt. Wie
hoch ist unter dieser Bedingung die Wahrscheinlichkeit, dass es schneien wird?
(g) Am Abend gab es keine andere Möglichkeit, als die Zeit mit Spielen
zu vertreiben. Peter meinte zu Paul: „Wir setzen beide einen Euro und
würfeln. Wer die höhere Zahl hat, gewinnt und bekommt beide Euro.
Sollten beide Würfel die gleiche Zahl zeigen, würfeln wir einfach gleich
noch einmal.“ Peter nahm einen normalen Spielwürfel, während Paul
ein Oktaeder mit den Zahlen 1 bis 8 in der Spielsammlung fand. Das
fand Peter weniger lustig. Wie muss man Peters und Pauls Einsatz
(auf Cent genau) ändern, damit das Spiel fair ist und die Summe der
Einsätze weiterhin 2 Euro beträgt?
12. Bedingte Wahrscheinlichkeit (? Punkte)
Von den in der Firma ElecExe hergestellten Computern sind erfahrungsgemäß 4% fehlerhaft. Bei der Endkontrolle wird ein fehlerhaftes Gerät mit
der Wahrscheinlichkeit 0, 8 beanstandet. Es kommt durchaus vor, dass ein
einwandfreies Gerät beanstandet wird, und zwar mit der Wahrscheinlichkeit
0, 1. Wie groß ist die Wahrscheinlichkeit dafür, dass
(a) ein bei der Endkontrolle beanstandetes Gerät auch tatsächlich fehlerhaft ist ?
(b) ein nicht beanstandetes Gerät fehlerfrei ist ?
13. Modellierung (? Punkte)
Zwei Brüder spielen: David hat fast nur Schach im Kopf und nervt seine
Umgebung mit ständigem Bitten um das Spielen einer Partie. Um vor ihm
etwas Ruhe zu haben, hat sein großer Bruder Philipp versprochen, mit ihm
drei Partien zu spielen. Vorläufig ist er noch der stärkere Spieler: Er besiegt
David mit einer Wahrscheinlichkeit von 60% und wird nur mit 10% Wahrscheinlichkeit von ihm geschlagen. Die übrigen 30% aller Partien enden also
76
ANHANG A. ÜBUNGSAUFGABEN
„remis“ (= unentschieden). Zu berechnen ist die Wahrscheinlichkeit dafür,
dass
(a) David alle drei Partien verliert.
(b) David genau einmal gewinnt.
(c) David mindestens einmal gewinnt.
(d) Mindestens zwei der drei Partien remis enden.
Die Mutter der beiden Jungen hört im Nebenzimmer, wie nach der ersten
Partie „Schachmatt!“ gerufen wird.
(e) Wie groß ist jetzt die Wahrscheinlichkeit dafür, dass Philipp der Sieger
ist?
A.5
Übungen Vektoralgebra
Aufgabe A.20
1. Welche Bedingung müssen zwei Vektoren ~v , w
~ ∈ Rn mit
Ursprung im Nullpunkt (den durch ~0 festgelegten Punkt) erfüllen, damit
folgendes gilt: (~v + w)⊥(~
~
v − w),
~ d.h. (~v + w)
~ orthogonal zu (~v − w)
~ ?
2. Gegeben sei ein beliebiger Vektor ~v ∈ Rn . Als Normierung des Vektors ~v
bzw. Einheitsvektor bezeichnet man den Vektor ~v 0 , der die gleiche Richtung
wie ~v hat, jedoch die Länge 1 besitzt, d.h. ||~v 0 || = 1. Geben Sie eine allgemeine Transformationsvorschrift an, die einen beliebigen Vektor normiert
bzw. in seinen ihm zugehörigen Einheitsvektor überführt. Zeigen Sie, dass
die Transformation korrekt arbeitet.
Eine Transformation, die die Länge eines Vektors verfünfacht wäre z.B. die
Multiplikation des Vektors mit 5:
p
p
p
5 ∗ ||~v || = 5 ∗ < ~v , ~v > = 25∗ < ~v , ~v > = < 5 ∗ ~v , 5 ∗ ~v >
3. Es sei eine Basis (~bi )i∈I gegeben. Ein gegebener Vektor ~v ∈ Span(~bi )i∈I kann
nun durch seine Komponenten bzw. Koordinaten ~v~bi (jeweils in Richtung
von ~bi ) linear kombiniert werden, d.h.
X
~v =
~v~bi
i∈I
Wie können die ~v~bi berechnet werden ?
Es seien die (~bi )i∈I nun Einheitsvektoren, d.h. ||~bi || = 1, mit der zusätzlichen
Bedingung: Wenn ~bi = (bi1 , . . . , bin ), dann gelte bii = 1 und sonst bij = 0. Wie
sehen in diesem Fall die ~v~bi aus ?
A.5. ÜBUNGEN VEKTORALGEBRA
77
4. Untersuchen Sie die folgenden Vektoren auf lineare Unabhängigkeit bzw.
lineare Abhängigkeit:
(a) ~a = (1, 2, 3), ~b = (−1, 3, 1), ~c = (2, 5, 0)
(b) ~a = (2, 0, −1), ~b = (−1, 3, −4), ~c = (1, 9, −14)
(c) ~a = (1, 0, 1), ~b = (2, 1, −3), ~c = (−1, −1, 0)
5. Das Spatprodukt dreier Vektoren ~u, ~v , w
~ ∈ R3 mit ~v × w
~ 6= 0 ist definiert
durch:
Spat(~u, ~v , w)
~ =< ~u, ~v × w
~>
Berechnen Sie das Volumen V der von folgenden Vektoren aufgespannten
„Raumes“ mittels dem Spatprodukt!
(a) ~a = (1, 2, 3), ~b = (−1, 3, 1), ~c = (2, 5, 0)
(b) ~a = (1, 0, 1), ~b = (2, 1, −3), ~c = (−1, −1, 0)
6. Stellen Sie für die Vektoren aus der vorangegangenen Aufgabe jeweils die
Determinante zu folgendem Gleichungssystem auf und bestimmen Sie jeweils ihren Wert! x1 ∗ ~a + x2 ∗ ~b + x3 ∗ ~c = ~0
Bestimmen Sie dann jeweils die Werte für die xi .
Gibt es einen Zusammenhang zu den vorangegangenen Aufgaben (außer,
daß die gleichen Vektoren verwendet wurden) ?
Gilt dieser Zusammenhang auch für die Vektoren ~a = (2, 0, −1), ~b = (−1, 3, −4), ~c =
(1, 9, −14) ?
Aufgabe A.21
1. Es seien ~x = (−2, 3, −4), ~y = (a, 12, b). Bestimmen Sie die
Zahlen a und b so, daß es ein r ∈ R gibt mit ~y = r ∗ ~x.
2. Es seien ~x = (−2, 3, −4), ~y = (a, 12, b). Bestimmen Sie die Zahlen a und b
so, dass < ~y , ~x >= 8.
3. Bestimmen Sie jeweils alle möglichen Lösungen für die Skalare xi .
(a) x1 ∗ (2, 0) + x2 ∗ (−3, 1) = ~0
(b) x1 ∗ (4, 0, 0) + x2 ∗ (6, −3, 0) + x3 ∗ (4, 2, 1) = ~0
(c) x1 ∗ (1, 0) + x2 ∗ (1, −11) + x3 ∗ (4, 3) = ~0
(d) x1 ∗ (4, −6) + x2 ∗ (10, −15) = ~0
Aufgabe A.22 Überprüfen Sie, ob die Vektoren des R2 bzw. des R4 liner abhängig oder linear unabhängig sind.
1. (4, −6); (10, −15)
78
ANHANG A. ÜBUNGSAUFGABEN
); (−9, 3, −12, 3)
2. (2, −8, 6, 0); (1, 32 , 21 , −1
2
Aufgabe A.23 Gegeben seien zwei Vektoren ~a und ~b. Wir setzen ~c := ~a + ~b.
Wann sind die beiden Vektoren ~a und ~c linear abhängig bzw. linear unabhängig
(bzgl. der Gestalt der beiden Vektoren ~a und ~b) ?
Aufgabe A.24 Zeigen Sie , daß W := Span((−2, 7, 1); (2, 5, 3)) ein Untervektorraum von v := Span((−2, 7, 1); (2, 5, 3); (0, 12, −5)) ist.
Aufgabe A.25 Prüfen Sie nach, ob folgende Mengen Untervektorräume (UVR)
der angegebenen Vektorräume (V) sind:
1.
{~x|~x = (x1 , x2 , x3 ) und x1 + x2 = 0}; V = R3
2.
{~x|~x = (x1 , x2 ) und x1 = 0; x2 ≥ 0}; V = R2
Aufgabe A.26 Bezüglich eines Skalarproduktes gelte für zwei Vektoren ~a, ~b: <
~a, ~a >= 6, < ~a, ~b >= 3, < ~b, ~b >= 2. Berechnen Sie:
1. < ~a + ~b, ~a >
2. < ~a + ~b, ~b >
3. < 3 ∗ ~a − ~b, 2 ∗ ~a + 5 ∗ ~b >
4. < x1 ∗ ~a + x2 ∗ ~b, ~a + ~b >
A.6
Übungen Matrizenalgebra
Aufgabe A.27 Eine n × m Matrix A wird im folgenden geschrieben als An,m .
Gegeben seien nun die Matrizen A3,4 , B3,4 , C4,2 , D3,2 , E4,3 . Entscheiden Sie, welche
der folgenden Matrizenberechnungen definiert sind und geben Sie das Format (n×
m) der resultierenden Matrix an bzw. begründen Sie, warum sie nicht definiert
ist!
a) B∗A
b) A∗C+D
c) A∗E+B
d) E∗(A∗C)
e) (TA+E)∗D
f ) C+T(E ∗ D)
Aufgabe A.28 Berechnen Sie die Produkte A∗B, A∗ TB, TA∗B, TA∗ TB, B ∗A,
B ∗ TA, TB ∗ A, TB ∗ TA falls diese existieren für
 
1
A =  2  eine 3 × 1Matrix und B = (3 2 1) eine 1 × 3Matrix.
3
A.6. ÜBUNGEN MATRIZENALGEBRA
79
Aufgabe A.29 Bestimmen Sie die Lösung(-en) des folgenden Gleichungssystems.
Beachten Sie dabei, dass bei diesem linearen Gleichungssystem keine eindeutige
Lösung existiert, sondern „nur“ ein Lösungsraum!
1.
 


 
x1
3
−13 20 −2
10

 0, 2

 
0
6 0, 8 
 ∗  x2  =  −2 

 0, 4 −1, 3 5 0, 2   x3   0 
−1, 2 5, 2 −8 0, 8
x4
−4
2.
3.
 
x1
1, 5 −6, 5 10 −1


 0, 5
0
15 2   x2

 0, 4 −1, 3 5 0, 2  ∗  x3
−0, 6 2, 6 −4 0, 4
x4


5
  −5 

=
  0 
−2



 
 
6, 4
x1
3, 2 −4, 8 14, 4
 −0, 6 0, 9 −2, 7  ∗  x2  =  −1, 2 
8
x3
4
−6
18

Aufgabe A.30 Zeigen Sie, dass für eine n × n Matrix A die Beziehung
det(A−1 ) = (det(A))−1
gilt.
Tip: Gehen Sie von A ∗ A−1 = E aus und verwenden Sie det(A ∗ B) =
det(A) ∗ det(B).
Aufgabe A.31 Gegeben sei die folgende Matrix A:
 1

1
1

A=
√
3
√1
3
√1
3
√
2
0
−1
√
2
√
6
−2
√
6
√1
6


1. Bestimmen Sie die transponierte Matrix TA.
2. Bestimmen Sie A ∗ TA.
3. Bestimmen Sie die Inverse A−1 .
Aufgabe A.32 Gegeben sei die folgenden Matrizen A1 , A2 :


1 0 1
a b
A2 =  2 2 2 
A1 =
c d
4 1 3
80
ANHANG A. ÜBUNGSAUFGABEN
1. Bestimmen Sie die transponierten Matrizen TA1 , TA2 .
2. Bestimmen Sie A1 ∗ TA1 , A2 ∗ TA2 .
3. Bestimmen Sie die Inverse A−1
1 . Testen Sie Ihr Ergebnis durch Bestimmung
−1
von A1 ∗ A1 .
Aufgabe A.33
1. Gegeben sei folgende Matrix:

0 0 1 −1
 0 0 −1 1

0
M cCP = 
 0 0 0
 0 0 0
0
0 0 0
0
0
0
1
1
0






Gegeben seien weiter die vier Vektoren
x~1 = (0, 0, 0, 0, 0); x~2 = (0, 1, 0, 1, 0); x~3 = (1, 0, 1, 0, 1); x~4 = (1, 1, 1, 1, 1);
Gegeben sei folgende Funktion auf den Vektoren: ΛΘ (~x) := (λΘ (xi ))i
1 falls xi ≥ Θ
λΘ (xi ) :=
0 falls xi < Θ
Hier ist Θ = 0.5. Dann ist z.B. ΛΘ ((−1, 0, 2, 1, −2)) = (0, 0, 1, 1, 0).
Multiplizieren Sie nun in folgender Weise die

0
 0

x~i ∗ M cCP = (xi1 , xi2 , xi3 , xi4 , xi5 ) ∗ 
 0
 0
0
Vektoren mit der Matrix:

0 1 −1 0
0 −1 1 0 

~i
0 0
0 1 
=y
0 0
0 1 
0 0
0 0
Führt man diese Multiplikation allgemein durch, erhält man:
y~i = (0, 0, xi1 + xi2 ∗ (−1), xi1 ∗ (−1) + xi2 , xi3 + xi4 ) =
(0, 0, xi1 − xi2 , xi2 − xi1 , xi3 + xi4 )
Führen Sie nun für jeden Vektor x~i folgenden Algorithmus durch:
Algorithmus A.3 (a) ~y 0 := x~i ;
(b) Für j := 1 bis 2 tue: ~y j := ΛΘ (~y j−1 ∗ M cCP );
Dokumentieren Sie die Anwendung des Algorithmus nachvollziehbar.
Vergleichen Sie nun die xi1 , xi2 mit den y5i des Vektors ~y 2 , der bei der letzten Berechnung des Algorithmus entsteht. Welche Auswirkung haben die
xi3 , xi4 , xi5 auf den jeweiligen Vektor ~y 2 ? Begründen Sie Ihre Antworten!
A.7. ÜBUNGEN DETERMINANTEN
A.7
81
Übungen Determinanten
Aufgabe A.34 Bestimmen Sie die Lösung des folgenden Gleichungssystems mit
der Cramer’schen Regel:
2x1 − x2 + 3x3 = 9
x1 + x2 + x3 = 2
−5x1 − 2x2
= −3
Aufgabe A.35 Bestimmen Sie die Lösung des folgenden Gleichungssystems mit
der Cramer’schen Regel:
x1 + x2
= 1
x2
+ x3 = 1
3x1 + 2x2 + x3 = 0
Aufgabe A.36 Beweisen Sie nachfolgende Gleichung:
x +x y1 +y2
1 2
1
2
2
x −x
1 x1 y1 y
−y
1
2
1 = ∗ D = 1 2 2
2
x2 y2 2
x1
y1
1
Aufgabe A.37 Für welche k ∈ R ist folgendes
?
3x1 + 2x2 + 2x3
x1 + x2 + 3x3
3x1
+ x3
lineare Gleichungssystem lösbar
= 7
= k
= 0
Aufgabe A.38 Berechnen Sie die folgende Determinante:
1
2 0 −1
2 0
1 1
2
0 1 −1 0
1
3 2
1 1
0
0 1
2 1 −1 −1 
Herunterladen